Exam 4 Pop health

Lakukan tugas rumah & ujian kamu dengan baik sekarang menggunakan Quizwiz!

A nurse examining a child in the early stages of HIV infection would expect to see: a. Failure to thrive and developmental delays b. Kaposis sarcoma and developmental delays c. Toxoplasmosis and oral candidiasis d. Fatigue and shortness of breath

ANS: A Early symptoms of pediatric HIV infection include failure to thrive and developmental delays.

Virtue ethics is distinctly different from moral justification via theories or principles because the emphasis of virtue ethics is practical reasoning applied to: A. Character development. B. Consequentialism. C. Distributive justice. D. Egalitarianism.

A. Character development. The goal of virtue ethics, one of the oldest ethical theories, is to enable individuals to flourish as human beings. According to Aristotle, virtues are acquired, excellent traits of character that dispose humans to act in accordance with natural good. Examples of such traits in virtue ethics are benevolence, compassion, discernment, trustworthiness, integrity, and conscientiousness.Cognitive Level: AnalysisAssociated Chapter Objective: 3Page Reference: 131

The community oriented nurse best exemplifies the application of feminist ethics when the nurse: A. Designs health care programs for the underserved that incorporate social justice, treat men and women with respect and equity, and include community interventions that elevate the status of the underserved in the community. B. Designs health care programs for women that respect their dignity and autonomy. C. Ensures that male providers do not use sexist terms when counseling clients and their spouses and when dealing with female colleagues. D. Participates in political action committees that focus on women's rights and status in the community.

A. Designs health care programs for the underserved that incorporate social justice, treat men and women with respect and equity, and include community interventions that elevate the status of the underserved in the community. Whereas feminism rejects the devaluing of women and their experiences, believes that the oppression of women is morally wrong, and demands social justice and political action, feminist ethics holds a broader worldview advocating just relationships for both men and women, adopts a feminist perspective that facilitates critical thought, and focuses on broad issues such as power, gender, and socioeconomic structures.Cognitive Level: AnalysisAssociated Chapter Objective: 3Page Reference: 132-133

One step in the ethical decision-making framework is to place an ethical issue within a meaningful context. The rationale for this step is that: A. Multiple factors affect the formulation and justification of ethical issues and dilemmas. B. The nature of ethical issues and dilemmas determine the specific ethical approach used. C. People cannot make sound ethical decisions if they cannot identify ethical issues and dilemmas. D. Professionals cannot avoid choice and action in applied ethics.

A. Multiple factors affect the formulation and justification of ethical issues and dilemmas. The historical, sociological, cultural, psychological, economic, political, communal, environmental, and demographic contexts affect the way ethical issues and dilemmas are formulated and justified.Cognitive Level: AnalysisAssociated Chapter Objective: 2Page Reference: 127-129

A visitor from Japan comes to the United States for a two-week vacation. This person is known as a: a. Non-immigrant b. Refugee c. Legal immigrant d. Lawful permanent resident

ANS: A A non-immigrant is admitted for a limited duration for a specific purpose.

The relationship between nursing practice, health policy, and politics can best be described as nursing: A. advocacy. B. policy process. C. process. D. profession.

ANS: A Advocacy begins with the art of influencing others (politics) to adopt a specific course of action (policy) to solve a societal problem and is accomplished by building relationships with appropriate policy makers—the individuals or groups that determine a specific course of action to be followed by a government or institution to achieve a desired end (policy outcome). This can be done in many ways. Types of advocacy include actions on behalf of clients served by nursing, especially vulnerable populations; activities of the nursing profession itself; work to develop needed health policies or revise existing policies; and actions related to the community.

Research has shown that Asian men tend to have a greater sensitivity than white Europeans to codeine, and they experience significantly weaker effects from the drug. This is an example of cultural variations of: a. Biological variations b. Space c. Social organization d. Time perception

ANS: A Biological variations are the physical, biological, and physiological differences that exist between racial groups and distinguish one group from another.

One effect of the use of block grants was a: a. Reduction of federal expenditures b. Decreased ability of states to spend money on programming c. Guaranteed continuation of programs with demonstrated effectiveness d. Shift from hospital-based to community-based nursing care

ANS: A Block grants enabled financial responsibility to shift from the federal level to the state level, thereby reducing expenditures.

The state orders a person newly diagnosed with tuberculosis to be quarantined. What type of law allows the state to do this? a. Constitutional b. Common c. Judicial d. State

ANS: A Constitutional law provides overall guidance for selected practice situations.

To meet a clients needs, it is sometimes necessary to integrate into the clients care a culturally relevant practice that lacks scientific utility. This is known as cultural: a. Accommodation b. Brokering c. Preservation d. Repatterning

ANS: A Cultural accommodation refers to assistive, supportive, facilitative, or enabling nurse actions and decisions that help people of a particular culture to accept nursing strategies, or to negotiate with nurses to achieve satisfying health care outcomes.

When a local health department provides a hypertension screening to all individuals, which general category of health care functions is being used? a. Direct services b. Financing c. Information d. Policy setting

ANS: A Direct services are actions that provide health care directly to individuals.

A nurse believes that the best treatment for illness is the use of Western medicine and alternative therapies should not be used for healing. This is an example of: a. Ethnocentrism b. Cultural imposition c. Racism d. Stereotyping

ANS: A Ethnocentrism is the belief that ones own cultural group determines the standards by which another groups behavior is judged.

An example of an explicit cultural behavior is: a. Verbal communication b. Body language c. Use of titles d. Perception of health and illness

ANS: A Explicit behaviors are straightforward and do not leave room for misinterpretation of what the person wants to communicate, such as when using verbal communication. Implicit behaviors are less exact and may be difficult to interpret, including body language, use of titles, and perception of health and illness.

HIV transmission can occur through: a. Exposure to blood b. Insect bites c. Sharing of school supplies d. Toilets

ANS: A HIV can be transmitted through exposure to blood.

A nurse is appraising health risks. Which of the following questions would most likely be asked by the nurse? a."Does your 4 year-old have a booster seat in the car?" b."Have you noticed any physical problems as you go about your daily routine?" c."What concerns do you have today? d."Why did you decide to come in for a checkup?"

ANS: A Health risk appraisal refers to the process of assessing for the presence of specific factors in each of the categories that have been identified as being associated with an increased likelihood of an illness, such as cancer, or an unhealthy event, such as an automobile accident

Which type of hepatitis would likely be found where sanitation is inadequate? a. A b. B c. C d. D

ANS: A Hepatitis A would likely be found where sanitation is inadequate.

A nurse is uncomfortable discussing such topics as sexual behavior and sexual orientation when counseling clients. By avoiding this topic with clients: a. Potential risks and risky behaviors will not be identified. b. Transmission of sexually transmitted diseases will decrease. c. Clients will be reluctant to return to care providers. d. The nurse will be violating the laws in most states.

ANS: A It is important that nurses be able to discuss these topics to help prevent and control STDs. Without discussion of these topics, it is possible that clients will not be aware that they have an STD and may transmit it to others.

A nurse is assisting an employer who has hired an individual who has been recently diagnosed with HIV. Which intervention would be most appropriate for the nurse to implement? a. Educate about how to reduce the risk of breaching the employees confidentiality. b. Explain how to inform co-workers about avoiding HIV transmission. c. Facilitate obtaining medical insurance coverage for the HIV-infected employee. d. Describe the early signs and symptoms of HIV infection.

ANS: A Nurses frequently work in the education role, and employers may need assistance in dealing with HIV-infected employees.

It is important that nurses are involved in health policy because: a. Government and policy have a large impact on nursing and health. b. Policy affects nursing values as set forth by Florence Nightingale. c. Political science is a course of study that parallels nursing. d. Nurses must interpret laws to fit their practice.

ANS: A Nurses should be advocates for the health of the population. In order to do this, nurse professionals must have a working knowledge of government, health care law, the policy process, and the political forces that are shaping the future of health care.

Nurses who strive to be culturally competent should: a. Respect individuals from different cultures and value diversity. b. Immerse themselves in different cultures. c. Design care for special ethnic groups. d. Give explicit instructions to avoid client decision making.

ANS: A Nurses who strive to be culturally competent respect individuals from different cultures and value diversity. The other answers are false.

An occupational health nurse maintains a log of injuries from contaminated sharps because of the: a. Bloodborne Pathogen Standard b. Occupational Safety and Health Administration (OSHA) c. Department of Labor (DOL) d. Department of Defense (DOD)

ANS: A OSHA is part of the Department of Labor. The Bloodborne Pathogen Standard became effective in 2002.

A public health nurse is reviewing Healthy People 2020 to determine where to prioritize programming for the county health department. Based on Healthy People 2020, the nurse decides to implement programming to: a. Reduce the rate of HIV transmission among adults and adolescents. b. Eliminate STDs from developed countries. c. Reduce deaths from gonorrhea. d. Increase awareness about HIV in lesbian females.

ANS: A One of the Healthy People 2020 objectives is reducing the number of cases of HIV infection among adults and adolescents.

Pelvic inflammatory disease (PID) is a common complication of: a. Gonorrhea b. Syphilis c. Chancroid d. Herpes

ANS: A PID is a common complication of gonorrhea.

A nurse discusses with legislators the importance of passing legislation to ban smoking in all public places. This demonstrates the nurses involvement in: a. Policy b. Politics c. Law d. Health policy

ANS: A Politics is the art of influencing others to accept a specific course of action. Therefore political activities are used to arrive at a course of action (the policy).

Screening blood products and donor organs and tissue for hepatitis C infection is: a. Primary prevention b. Secondary prevention c. Tertiary prevention d. Health promotion

ANS: A Primary prevention refers to those interventions aimed at preventing the occurrence of disease, injury, or disability.

Several community-oriented nurses want to explore the problem of obesity in school-age children and assess their community school district's health status related to that problem. When gathering information at a national level, they would begin with the: A. Centers for Disease Control and Prevention. B. Centers for Medicaid and Medicare Services. C. Health Resources and Services Administration. D. National Institute of Nursing Research.

ANS: A The Centers for Disease Control and Prevention serve as the national focus for development and application of measures to advance disease prevention and control, environmental health, and health promotion, and for educational activities designed to improve the health of the people of the United States. The mission of the Centers for Disease Control and Prevention is to promote health and quality of life by preventing and controlling disease, injury, and disability.

The agency most heavily involved with the health and welfare concerns of United States citizens is the: a. Unites States Department of Health and Human Services (DHHS) b. Public Health Service c. Health Resources and Services Administration d. World Health Organization

ANS: A The DHHS is the agency most heavily involved with the health and welfare of United States citizens. It touches more lives than any other federal agency.

A member of the military receives health care services through which department? a. Defense b. Labor c. Agriculture d. Justice

ANS: A The Department of Defense delivers health care to members of the military, including their dependents and survivors, and to retired members and their families.

The nurse counsels a client to have the enzyme-linked immunosorbent assay (EIA) test in order to: a. Indicate the presence of the antibody to HIV. b. Reveal whether or not the client has AIDS. c. Isolate the HIV virus. d. Confirm HIV after having a positive Western blot.

ANS: A The EIA is used to indicate the presence of the antibody to HIV.

A nurse is working with a family member to reduce his health risk. Which of the following recommendations would most likely be made by the nurse? a."Be sure to take a 30-minute walk each day." b."Call our office if you have any questions or concerns at all." c."Come back in 2 weeks for follow-up on your surgery." d."Continue to take the drug until it is gone, even if you're feeling better earlier."

ANS: A The factors that determine or influence whether disease or other unhealthy results occur are called health risks. The major categories of risk include inherited biological risk, social and physical environmental risk, and behavioral risk. All the other options are treatment oriented rather than risk avoidance. Exercising for 30 minutes a day reduces the risk for many diseases

When working with an immigrant population, the first step the nurse should take is: a. Be aware of ones own culture. b. Become familiar with traditional practices of the immigrants. c. Try to see things from the immigrants viewpoint. d. Learn to speak the language of the immigrant population.

ANS: A The first statement is the only one that is correct.

When a school nurse reports suspected child abuse, there is a breach of confidentiality. This breach is allowed because of: a. Civil immunity b. Sovereign immunity c. Nurse practice act d. Respondeat superior

ANS: A The law mandates that a health professional breach patient confidentiality norms to protect someone who may be in a helpless or vulnerable position.

A nurse has been successful in creating improvement in a family's health. Which of the following characteristics is most likely displayed by the nurse? a.Skilled at recognizing and strengthening the family's competencies b.Skilled at obtaining referrals and resources for the family c.Skilled at communication and interpersonal relationships d.Skilled at assessing and naming the family's main problems

ANS: A The nurse's approach to the family should be positive and focused on competencies rather than on problems or deficits.

Which statement about the nursing workforce is true? a. The nursing workforce is overwhelmingly Caucasian. b. The number of minority nurses meets the needs of the countrys demographics. c. The nursing workforce has many unauthorized immigrants. d. Refugees make up a large part of the nursing workforce in some areas.

ANS: A The nursing work force is 81.8% Caucasian. The other statements are false.

Nurses are available to provide health services for women and children due to the provisions of the: a. Sheppard-Towner Act b. Public Protection of Maternity and Infancy program c. Early Periodic Screening and Developmental Testing (EPSDT) program d. Women, Infants, and Children (WIC) program

ANS: A This act also offered well-child and child-development services, provided adequate hospital services and facilities for women and children, and provided grants-in-aid for establishing maternal and child welfare programs.

The nurse labels a patient an alcoholic because of his ethnicity. This is called: a. Stereotyping b. Prejudice c. Racism d. Ethnocentrism

ANS: A This is an example of ascribing certain beliefs and behaviors about a given racial and ethnic group to an individual without assessing for individual differences.

A Mexican American mother insists on using special candles to help her daughters ear infection. The nurse asks the mother if she would also give her daughter antibiotics. This is called cultural: a. Accommodation b. Repatterning c. Brokering d. Awareness

ANS: A This means that the nurse supports and facilitates the use of cultural practices with interventions from the biomedical health care system. Cultural accommodation refers to assistive, supportive, facilitative or enabling nurse actions and decisions that help clients of a particular culture accept nursing strategies, or negotiate with nurses to achieve satisfying health care outcomes.

A nurse has completed health risk appraisals with several different families. Which of the following families would be of most concern to the nurse? a.An older couple who has just retired and sold their house, who talk about their new condo in a retirement community. b.Newlyweds who have been saving their money, who want to discuss birth control and family planning in preparation for future pregnancies. c.Parents who come with their child for his pre-kindergarten physical exam and want to be sure all the child's immunizations are up to date. d.A woman who is very pleased with her new position at the hospital and wants to have her pre-employment exam and drug screen

ANS: A Transitions (movement from one stage or condition to another) are times of potential risk for families. Age-related or life-event risks often occur during transitions from one developmentalstage to another. Transitions present new situations and demands for families. Moving from the family home to a smaller condo represents a major change in lifestyle. None of the other options represent major transitions. If the event is normative, or anticipated, it is possible for families to prepare for the event and its consequences.

A large portion of foreign-born residents of the United States: a. Work in service-producing and blue-collar sectors b. Reside in rural areas c. Have language barriers d. Are refugees and non-immigrants

ANS: A Two thirds of the foreign-born population lives in or around major metropolitan areas and works in mainly service-producing and blue-collar sectors. They are also more likely to be poorer. The majority of foreign born are legal immigrants (85%).

A nurse explained to a new mother that because she had tested positive for the hepatitis B virus, her newborn son would need the hepatitis B vaccine immediately and then also an immune globulin injection. "Wait," said the new mother. "Why is my son getting two shots?" Which of the following statements would be the best response by the nurse? a. "One injection protects your son, while the other encourages his body to build up immunity." b. "One shot keeps your son from getting sick, while the other is a typical vaccine to prevent you from accidentally infecting him." c. "Since you've already been infected with the virus, your son needs twice as much protection." d. "The second shot is just to make sure the first one works."

ANS: A Because infected persons may not have any symptoms, all pregnant women should be tested for HBsAg. If the mother tests positive, her newborn needs hepatitis B immune globulin to provide passive immunity and thus prevent infection. In addition, the newborn is given the hepatitis B vaccine at birth, with two follow-up injections, to build active immunity to the infection. One of the shots provides passive immunity and the other provides active immunity. The active immunity continues to be built up by receiving two follow-up injections weeks later.

A school nurse asks a class about the ways HIV can be transmitted. Which of the following comments by a student indicates a need for additional teaching? a. "I wouldn't sit next to someone with HIV." b. "Having unprotected sex with someone who is infected spreads HIV." c. "Sharing needles when shooting up drugs spreads HIV." d. "Transfusions of blood products that are contaminated can spread HIV."

ANS: A HIV is not transmitted through casual contact such as touching or hugging someone who has HIV infection. It is not transmitted by insects, coughing, sneezing, office equipment, or sitting next to or eating with someone who has HIV infection. HIV can be transmitted through exposure to blood, semen, vaginal secretions, and breast milk and by sharing needles, syringes, and other equipment used to prepare injectable drugs. It can also be spread by perinatal transmission from mother to child through delivery or breastfeeding and by transfusions of contaminated blood. Worldwide, the largest number of HIV infections result from heterosexual transmission

A male client visits the clinic office reporting a yellow-green discharge from his penis. Which of the following STIs has the client most likely contracted? a. Gonorrhea b. Syphilis c. Herpes simplex virus 2 d. Human papillomavirus

ANS: A The symptoms for gonorrhea in a male include a burning sensation when urinating, or a white, yellow-green discharge from the penis. Some men may get swollen or painful testicles. In men, gonorrhea can cause epididymitis, a painful condition of the testicles that if untreated can lead to infertility. Symptoms of syphilis may not appear for several years after contracting the disease. The first stage of syphilis is called primary syphilis when a chancre, a firm, round, small and painless lesion, develops. Signs and symptoms of HSV-2 infection range from no symptoms to painful lesions or blisters around the genitals, rectum, or mouth. Most people with HPV are asymptomatic.

Which of the following statements best explains why HSV-2 infection is more challenging for a client than gonorrhea infection? a. HSV-2 is a viral infection that is both chronic and incurable. b. HSV-2 is extremely expensive to treat. c. HSV-2, like HIV, is almost impossible to diagnosis in the early stages. d. Once a person has been treated for HSV-2, the person is immune to further outbreaks.

ANS: A Unlike gonorrhea, there is no cure for HSV-2 infection; it is considered a chronic disease. Recurrence with HSV-2 is common. The treatment for HSV-2 may be episodic or suppressive for frequent recurrence, which is not expensive to treat. HSV-2 may be asymptomatic but does appear as vesicles, painful ulceration of penis, vagina, labia, perineum, and anus with lesions lasting 5 to 6 weeks.

A client is using a primary prevention strategy to prevent infectious disease. Which of the following actions is the client most likely taking? a. A client receives a tetanus booster every 10 years. b. A client receives a tetanus booster after stepping on a nail. c. A client receives tetanus immunoglobulin after stepping on a nail. d. A client with tetanus is given antibiotics and is placed on seizure precautions.

ANS: A - A client receives a tetanus booster every 10 years. Tetanus boosters given before exposure are a measure of primary prevention because exposure has not yet occurred. If given after exposure (i.e., the client may be infected but disease has not developed), they are considered secondary prevention (similar to the textbook examples of immunoglobulin and rabies immunizations given after exposure). Immunoglobulin would be given if the client had not been previously immunized; however, this again is after exposure, so it is secondary prevention. Because the client has the condition, treatment is aimed at prevention of further injury.

Which of the following is the most probable cause of the increase in new emerging infectious diseases? a. Activities or behavior of humans, including changes in the environment b. Increasing urbanization and growth in new housing materials c. New infectious agents are evolving throughout the world d. Overpopulation in many areas, creating a need to reduce global population

ANS: A - Activities or behavior of humans, including changes in the environment Most of the emergence factors are consequences of activities and behavior of the human hosts and of environmental changes such as deforestation, urbanization, and industrialization. For example, the rise in households with two working parents has increased the number of children in daycare, and with this shift has come an increase in diarrheal diseases such as shigellosis. Urbanization is not a problem, but increasing development into formerly unaffected areas such as rainforests is.

Which of the following best represents an example of infectious disease spreading via a vector? a. Being bitten by an infected mosquito b. Disease spreading from infected mother to infant via the placenta c. A group of partygoers hugging and shaking hands d. Two persons, one of whom is infected, sharing a glass of soda

ANS: A - Being bitten by an infected mosquito Vertical transmission is the passing of infection from parent to offspring via placenta. Horizontal transmission is the person-to-person spread of infection through (among other ways) contact. Common vehicle refers to transportation of the infectious agent from an infected host to a susceptible host via food, water, milk, or other substance. Vectors include mosquitoes, which can transmit the infectious agent by biting the host.

A nurse's Mantoux test is positive for exposure to tuberculosis. Which of the following conclusions should be drawn by the nurse? a. The nurse has been exposed to tuberculosis. b. The nurse has tuberculosis. c. The positive test result probably is due to a problem in the testing process. d. The test is inaccurate and needs to be repeated.

ANS: A - The nurse has been exposed to tuberculosis. An individual who tests positive has been exposed and may be infected, but if that person shows no clinical signs, the person is not diseased. Infection refers to the entry, development, and multiplication of the infectious agent in the susceptible host. Disease is one of the possible outcomes of infection. People with latent TB have no symptoms, are not infectious, and can continue on with life. They may develop active TB. A nurse who interacts with clients may receive INH for a year as a precaution.

Which of the following biological warfare agents poses the greatest bioterrorism threat to a community? a. Anthrax b. Botulism c. Smallpox d. Tularemia

ANS: A - anthrax Because of factors such as the ability to become an aerosol, the resistance to environmental degradation, and a high fatality rate, inhalational anthrax is considered to have an extremely high potential for being the single greatest biological warfare threat.

Which of the following places best describes where the incidence of Vancomycin-resistant Staphylococcus aureus (VRSA) and methicillin-resistant S. aureus (MRSA) is currently rising? a. Areas where people share dressing or bathing facilities b. Daycare centers and schools c. Long-term care facilities d. Senior citizen centers

ANS: A - areas where people share dressing or bathing facilities Vancomycin-resistant Staphylococcus aureus (VRSA) and methicillin-resistant S. aureus (MRSA) remain problems for people who acquire the bacteria in the hospital, but there is a growing incidence of community-acquired MRSA in places where people closely share facilities such as locker rooms, prisons, and other close bathing areas.

1. Which of the following best describes the steps in program management? A. Assess, plan, implement, evaluate B. Identify, initiate, implement C. Organize, operationalize, mobilize, subsidize D. Substantiate, negotiate, evaluate

ANS: A Assess, plan, implement, evaluate The program management process is similar to the nursing process. Program management consists of assessing, planning, implementing, and evaluating a program. DIF: Cognitive Level: Remember (Knowledge) REF: pp. 276-277

9. A nurse is planning a program to teach cardiac health at the senior citizens center. Which of the following is an effectively written objective for the program? A. By the end of the program each participant will report walking at least 30 minutes a day at least 5 days each week. B. By the end of the program each participant will voice a commitment to walk at least 30 minutes a day. C. By the end of the program each participant will understand the need for physical exercise. D. Each participant will voice a commitment to engage in physical exercise each day.

ANS: A By the end of the program each participant will report walking at least 30 minutes a day at least 5 days each week. Useful program objectives must include a statement of the specific behaviors desired, using an action verb that can be seen and measured. Voicing a commitment is not an outcome action; it is only a verbal agreement. The verb understand is not an action verb that can be seen and measured. Only "will report walking 30 minutes a day at least 5 days each week" has a specific outcome action that can be seen and measured. DIF: Cognitive Level: Apply (Application) REF: p. 283

18. A nurse is conducting program evaluation. Which of the following would be the first action the nurse would take? A. Choose the type of evaluation to be done B. Determine who will be involved in the evaluation C. Identify the goal and objectives for the evaluation D. Obtain answers to specific questions related to the program being evaluated

ANS: A Choose the type of evaluation to be done To do a program evaluation, first choose the type of evaluation you wish to do. Second, identify the goal and objectives for evaluation. Third, decide who will be involved in the evaluation. Fourth, answer the questions related to the type of evaluation. DIF: Cognitive Level: Apply (Application) REF: p. 285

A woman and man who have come to the health clinic begin to argue loudly. "You gave me an STI!" the man yells. The woman screams back, "Not me. I don't have an STI!" Which of the following statements would be most appropriate for the nurse to say to them? (Select all that apply.) a. "Actually, you're very fortunate to have been tested, so you and your partner can begin treatment before more serious damage is done." b. "Some STIs may not have any symptoms, so you need to be tested for other conditions and treated if necessary." c. "Sometimes the test is inaccurate, so before getting too upset, you should ask to be tested again." d. "You may be able to get treatment from your pharmacist so you won't have to be embarrassed like this again." e. "Some STIs are spread in ways that are non-sexual in nature."

ANS: A, B, C Often cases of gonorrhea and chlamydia are asymptomatic, so treatment may not be sought and these infections are spread to others through sexual activity. Similarly, during latency, syphilis has no symptoms. It should be noted, however, that STI test results can sometimes be incorrect and the coexistence of other medical conditions may cause a false-positive test result. Having a partner retested, if the results were negative, would also suggest retesting the first person. The pharmacist must receive an order from the primary care provider in order to dispense a prescription.

Congress's legal base for actions in health care include (select all that apply): a. Providing for the general welfare b. Raising funds to support research c. Regulating commerce among the states d. Providing spending power

ANS: A, B, C, D The legal base for actions in health care includes providing for the general welfare, regulating commerce among the states, providing spending power, and raising funds to support the military.

A nurse is planning a refugee outreach clinic at the neighborhood health center. To provide culturally competent care, the nurse should consider that (select all that apply): a. Their own background, beliefs, and knowledge may be significantly different from those of the people receiving care. b. Language barriers may interfere with efforts to provide assistance. c. Certain risk factors may be present for a given population. d. Certain groups may use non-traditional healing practices.

ANS: A, B, C, D When working with immigrants it is important to be aware of ones own beliefs, realize that language barriers may exist, that different populations experience different risk factors and illnesses, and that non-traditional healing practices may be used.

State and federal statutes and regulations affect the health care specialties of home health and hospice practice. A primary motivator for nurses to become knowledgeable about these statutes and regulations is their impact on nursing practice in which of the areas below? (Select all that apply.) A. Documentation of client status and progress B. Documentation of services C. Living wills and advance directives D. Resident's rights in long-term care facilities E. Right to death with dignity

ANS: A, B, C, D, E State laws specify licensure and certification requirements for home health care and hospice agencies. Compliance with these laws is directly linked to the method of payment for the services. For example, a service must be provided by a licensed and state-certified agency to obtain payment from Medicare. Federal regulations implementing Medicare and Medicaid programs have an enormous effect on much of nursing practice, including documentation practices and recording of visits, client care, status, and progress. Other laws focus on issues such as the right to death with dignity, the rights of residents in long-term care facilities, definitions of death and death pronouncement, and the use of living wills and advance directives.

The Centers for Disease Control and Prevention accomplishes its mission by (select all that apply): a. Implementing prevention strategies b. Detecting and investigating health problems c. Dictating world health policy d. Fostering safe and healthful environments

ANS: A, B, D The Centers for Disease Control and Prevention implements prevention strategies, detects and investigates health problems, and fosters safe and healthful environments.

20. Which of the following are elements of the MAPP (Mobilizing for Action Through Planning and Partnership) Program Planning Model? (Select all that apply.) A. Generate shared visions and common values B. Assess priorities in health problems C. Develop a framework for long-range planning D. Choose health priorities E. Conducting a needs assessment related to community change

ANS: A, C A. Generate shared visions and common values C. Develop a framework for long-range planning The elements of MAPP include: mobilizing community members and organizations, generating shared visions and common values, developing a framework for long-range planning, conducting needs assessments in four areas: community strengths, local public health system, community health status, and focus of change, and implementing the plan. Assessing priorities in health problems and choosing health priorities are part of other program planning models. DIF: Cognitive Level: Remember (Knowledge) REF: p. 281

The nurse is counseling a female who has recently tested positive for HIV. The nurse educates her that she is responsible to (select all that apply): a. Have regular medical evaluations and follow-ups. b. Donate blood, plasma, body organs, or sperm. c. Inform health care providers about the HIV infection. d. Consider the risk of perinatal transmission.

ANS: A, C, D A person who is infected with HIV should have regular medical evaluations and follow-up appointments; not donate blood, plasma, body organs, or sperm; inform health care providers about the HIV infection; and consider the risk of perinatal transmission and follow-up with contraceptive use.

An example of a behavior that may be defined by culture is: a. Speaking a dialect of a language in a local region b. Standing when an older adult gets on the bus to give him a seat c. Immigrating to the United States seeking work d. An organizational structure of a cultural group

ANS: B Behaviors may be culturally determined as illustrated in the correct response. This behavior is based on the value of respect of elderly people.

An effect of providing care that is not culturally competent is: a. Enhancement of communication b. An increase in the cost of health care c. Achievement of health indicators d. Improvement in client compliance

ANS: B Care that is not culturally competent may increase health care costs and decrease positive client outcomes.

A nurse advocates and intervenes between the health care system and the clients cultural beliefs on behalf of the client. The nurses action is called cultural: a. Accommodation b. Brokering c. Preservation d. Repatterning

ANS: B Culture brokering is advocating, mediating, negotiating, and intervening between the clients culture and the biomedical health care culture on behalf of clients.

A public health nurse (PHN) has identified a need to make amendments to an existing law concerning the tuberculosis health assessment of individuals sentenced to serve jail terms on weekends only because of the gap in treatment accountability and potential health risk to the free-living community. To draw attention to this concern, the nurse has several paths to follow, but the amendment of any existing laws would ultimately be decided by which of the following? A. Executive branch of government B. Legislative branch of government C. Local representative D. Senate hearings

ANS: B Each of the government branches at the federal level, in most states, and at the local level plays an important role in developing and implementing health law and public policy. Concerned citizens have many avenues for addressing issues related to needed laws and regulations as well as existing laws and regulations. However, each branch of government has a separate and important function. The legislative branch identifies problems and proposes, debates, passes, and modifies laws to address identified needs.

A client shares that her grandparents immigrated to the United States from Germany. She has disclosed her: a. Multiculturalism b. Ethnicity c. Race d. Culture

ANS: B Ethnicity represents the identifying characteristics of culture, such as race, religion, or natural origin.

One of the main purposes of the Patient Protection and Affordable Care Act was to: a. Create a government-funded insurance program for all Americans. b. Make health insurance affordable for the middle class and small businesses. c. Decrease the rates of chronic disease in the United States. d. Increase the funding for public health agencies.

ANS: B One of the main purposes of this legislation was to make health insurance affordable for middle class families and small businesses with one of the largest tax cuts for health care in history: reducing premiums and out-of-pocket costs. This act does not create a government-funded insurance program nor does it increase funding for public health agencies.

The levels of government responsible for carrying out the five government health care functions of direct services, financing, information, policy setting, and public protection are: A. federal and state. B. federal, state, and local. C. all, but primarily state and local. D. all, but primarily federal and state.

ANS: B Federal, state, and local governments are responsible for carrying out the five core government health care functions. All levels of government provide direct services, but the targeted populations may vary; for example, at the federal level, the focus is on the members and dependents of the military, whereas the states focus on childhood immunizations. The federal government provides funding for health care education and research. The states provide maternal-child health care services through state-level funding for the poor or near poor. Local governments/organizations provide health care services through funding for free clinics that serve the uninsured. All branches and levels of government collect vital statistics and census data, and conduct health surveys. Governments, agencies, and organizations at all levels make health policy. Functions that protect the public, such as monitoring air and water and regulating food, drug, or animal transportation, are carried out at all levels. Affirmation of a woman's right to reproductive privacy occurred at the federal level (Supreme Court). Requiring vaccinations for school entry occurs at the state level.

Which of the following best describes the use of genomic health care? a.Assists with understanding family relationships b.Assists with determining familial health risks c.Useful in learning about environmental risk factors d.Useful in detecting risk for developing cancer

ANS: B Genomic health care can give health care providers the tools that they need to use a person's unique genomic information to design and prescribe the most effective treatment for each person and to help clients and families understand some of their health risks that are influenced by their genetic make-up. When nurses obtain a family history and learn about the illnesses and causes of death of biologically related family members, they can then learn aboutshared genes, environment and lifestyle behaviors that can increase a person's risks for the same diseases that other family members experienced. Genomic health care is broader than detecting risk for developing cancer

An example of HIV transmission is: a. Having contact with a HIV-positive individual who is coughing b. An infant receiving breast milk from a HIV-positive mother c. Receiving a mosquito bite while in Africa d. Being near a HIV-positive individual who is sneezing

ANS: B HIV can be transmitted through breast milk.

In comparison with HIV infection in adults, HIV infection in infants and children: a. Has the same signs and symptoms b. Has a shorter incubation period c. Has a longer survival period d. Is detected by using the same tests

ANS: B HIV infection in infants and children has a shorter incubation period.

Needle stick legislation clarified the role of employers to: a. Use Universal Precautions when dealing with all patients. b. Select safer needle devices as they became available. c. Provide needle disposal boxes. d. Incinerate all infectious waste including needles.

ANS: B Health care facilities by law have to select safer needle devices and involve employees in identifying and choosing the devices.

Health policy can best be defined as a set course of action to: A. administer public health care programs at the federal level for disaster preparedness, response, and recovery. B. obtain a desired health outcome for an individual, family, group, community, or society. C. support publicly funded health care programs at the local, state, and national levels. D. support health care development and research to improve the health status of citizens.

ANS: B Health policy is a set course of action to obtain a desired health outcome, either for individuals, families, groups, communities, or societies. Policies are made not only by governments but also by institutions such as a health department or other health agency, a family, or a professional organization. Politics plays a role in the development of such policies. Politics is found in families, professional and employing agencies, and governments. Therefore, political activities are used to arrive at a course of action (the policy). Policy is a settled course of action to be followed by a government or institution to obtain a desired end.

The health policy-making body of the World Health Organization (WHO) released a policy statement on nursing and midwifery in 2013. The important fact for nurses to understand about WHO policy statements is that such statements: A. apply only to underdeveloped countries. B. are guides for in-country initiatives and priorities. C. carry the weight of international law. D. provide mandates for in-country legislatures.

ANS: B In 1946, the WHO was created through a United Nations initiative as a special autonomous organization. The WHO provides services worldwide to promote health, cooperates with member countries in promoting their health efforts, coordinates collaborative activities between countries, and disseminates information on biomedical research and vital international statistics. The World Health Assembly (WHA) is the policy-making arm of the WHO and meets annually. The WHA's health policy work provides policy options for many countries of the world in their development of in-country initiatives and priorities. Although WHA policy statements are very important everywhere, they are guides and not law.

States have certain continuing education requirements for persons to renew a nursing license. This type of law is called: a. Constitutional law b. Legislation and regulation c. Judicial and common law d. Police power

ANS: B Licensing is regulated by each states nurse practice act as a function of the board of nursing.

An occupational health nurse is developing an educational program to address the importance of healthy personal health habits. Which of the following topics would be most important for the nurse to address? a.Avoidance of alcohol b.Regular physical exercise c.Daily consumption of calcium-rich foods d.Monthly self-breast and testicular exams

ANS: B Many family health risks can be reduced by careful attention to diet, exercise, and stress management. Regular physical exercise is effective in promoting and maintaining health and in preventing disease. Physical activity can help to prevent obesity, diabetes, heart disease, cancer, osteoporosis, and depression. Avoidance of alcohol and daily consumption of calcium-rich foods are not recommendations for improving personal health habits

A doctoral prepared nurse (PhD) at a university wants to submit a grant to study the impact of a nursing activity on patient outcomes. From which federal agency could this nurse potentially seek funding? a. Centers for Disease Control and Prevention (CDC) b. National Institute of Nursing Research (NINR) c. Department of Health and Human Services (DHHS) d. World Health Organization (WHO)

ANS: B NINR funds the development of the knowledge base of nursing and promotion of nursing services in health care.

A nurse was preparing for a home visit to a family where the mother had just been discharged from trauma care after being hit by a drunk driver. The nurse hoped the family was able to care for her. Which of the following comments from the husband would suggest an energized family? a."I make most of the decisions so the kids don't notice much difference." b."My daughter is pretty independent; she's active in both sports and theater." c."My son is old enough to get a job and help pay all these medical bills." d."My wife taught our daughter how to cook simple meals."

ANS: B Pratt proposed the energized family as being an ideal family type that was most effective in meeting health needs. The energized family is characterized by active contact with a variety ofgroups and organizations (Boy Scouts, church, sports, theater), flexible role relationships (not if only the daughter is taught how to cook and only the son is expected to get a job), equal power structure (not if the husband makes all the decisions), and a high degree of autonomy by each member (kids know how to be flexible, because parents travel for business frequently).

Teaching a client with gonorrhea how to prevent reinfection and further spread is an example of _____ prevention. a. Primary b. Secondary c. Tertiary d. Primary health care

ANS: B Secondary prevention focuses on early detection and prompt treatment of disease, injury, or disability.

The 1986 Immigration Reform and Control Act had what effect on illegal aliens living in the United States? a. Discriminated against individuals from Southern and Eastern Europe b. Allowed illegal aliens already living in the United States to apply for legal status c. Permitted foreign-born populations to set up communities in or around major metropolitan areas d. Allowed illegal aliens access to green cards that would allow them to work in the United States

ANS: B The 1986 Immigration Reform and Control Act permitted illegal aliens already living in the United States an opportunity to apply for legal status if they met certain requirements.

If a nurse researcher wants to get information on the occurrence of an internationally important disease, where would be a good place to seek this information? a. The United Nations website b. The World Health Organization (WHO) website c. The World Bank website d. The World Health Assembly website

ANS: B The WHO publishes day-to-day information about international occurrence of disease, injury, and death.

Which part of the immunological system suffers the greatest damage as a result of HIV infection? a. Dendrite cells b. CD4+ T-lymphocytes c. Macrophages d. Monocytes

ANS: B The count drops when the virus is most plentiful in the body.

The United States Department of Health and Human Services (DHHS) is part of which branch of the government? a. Judicial b. Executive c. Legislative d. Health

ANS: B The executive branch includes regulatory departments, one of which is the United States DHHS.

When planning, delivering, and financing responsibility for programs is shifted from the federal level to the state level, it is called: a. Evolution b. Devolution c. Block granting d. State administration

ANS: B The process of shifting the responsibility for planning, delivering, and financing programs from the federal level to the states is called devolution. This was a major effort during the Reagan administration.

A nurse utilizes the provisions created by the Ryan White Comprehensive AIDS Resource Emergency Act when performing which intervention? a. Increasing AIDS awareness in the community b. Determining available health care services for HIV-infected individuals c. Preventing the transmission of AIDS to children from their mothers d. Allowing persons in the final stages of HIV to die with dignity

ANS: B This act provides services including emergency services, services for early intervention and care, and drug reimbursement programs for HIV-infected individuals.

What information would be appropriate for a nurse to share when educating IV drug users about sharing equipment? a. Tell the clients to throw away their equipment after one use. b. Educate the clients on using full-strength bleach on their drug paraphernalia for 30 seconds. c. Report illegal activities to the authorities and confiscate the needles and syringes. d. Give out needles and syringes to whoever wants them.

ANS: B Using bleach on the needles and syringes is a way to decrease cross contamination.

Which of the following best explains why some health clinics allow clients to be tested for HIV anonymously with no record of the client's name, address, or contact information? a. Client doesn't actually ever have to be told the results of the test. b. Client may be engaged in illegal activities (drug use). c. Client plans on not paying for the test, and collection agencies will not be able to harass them. d. Client wants to be sure care providers don't share results with their family.

ANS: B An advantage of anonymous testing may be that it increases the number of people who are willing to be tested, because many of those at risk are engaged in illegal activities. The anonymity eliminates their concern about the possibility of arrest or discrimination. If testing is anonymous, the client is given an identification code number that is attached to all records of the test results and is not linked to the person's name and address. Demographic data such as the person's sex, age, and race may be collected, but there is no record of the client's name and associated identifying information. Thus, the client is able to know the results of the test while staying anonymous. It would be a violation of HIPAA for the providers to share the results of this test or any other medical information with family members. The follow-up related to payment is not a main concern for the population who is requesting anonymous testing.

A student complains to the college health nurse that her academic work has been going downhill because of lack of sleep. "My 3-year-old probably misses her babysitter since she has started going to the big daycare center. She hasn't been sleeping well and keeps scratching her bottom. Hopefully, she'll adapt to daycare soon." Which of the following information should the nurse provide to the student? a. "Dry skin in winter weather can cause itchiness; try to put on lotion before bedtime." b. "Your daughter may have pinworms; let me teach you how to check for this." c. "Perhaps your child is not developmentally ready for group play." d. "Try to arrange more one-on-one time with your 3-year-old."

ANS: B - "Your daughter may have pinworms; let me teach you how to check for this." Enterobiasis (pinworm infection) is the most common helminthic infection in the United States with about 42 million cases a year. This infection is seen most often among children in institutional settings. Pinworms cause itching, especially around the anus, which can result in a lack of sleep for both child and caregiver.

A mother felt very guilty that her baby was born HIV positive. When the nurse suggested the usual DPT and MMR immunizations, the mother was extremely upset. "Don't you know HIV children are immunosuppressed?" she exclaimed. Which of the following would be the nurse's best response? a. "All children have to have these immunizations before they can attend school." b. "Being HIV positive, your child is more likely to catch an infection and be very ill if not immunized." c. "I'm so sorry; I forgot for a moment your child was HIV positive." d. "The American Pediatric Association requires all health care providers to offer these immunizations to all parents; it is your choice whether or not to accept them."

ANS: B Because of impaired immunity, children with HIV infection are more likely to get childhood diseases and suffer serious consequences of the diseases. Therefore, DPT, IPV, and MMR vaccines should be given at regularly scheduled times for children infected with HIV. Other immunizations may also be recommended after medical evaluation. Although this child is HIV positive, there is no medical reason why the child should not be immunized. Many states do have a "no shots, no school" law, but exceptions can be made. The APA does recommend immunizations for most children, but this is not the best answer

Which of the following best describes the characteristic appearance of lesions of human papillomavirus (HPV)? a. Solitary growth with elevated borders and a central depression b. Elevated growths with a "cauliflower" appearance c. Thin-walled pustules that rupture to form honey-colored crusts d. Vesicles that ulcerate and crust within 1 to 4 days

ANS: B HPV causes genital warts that appear as textured surface lesions, with what is sometimes described as a cauliflower appearance. The warts are usually multiple and vary between 1 and 5 mm in diameter. The other descriptions are not characteristic of lesions of the human papillomavirus (HPV).

A client diagnosed with tuberculosis (TB) asks why the nurse is required to watch the client swallow the medication each day. Which of the following statements is the best response by the nurse? a. "Clients with TB are often noncompliant, so if I directly observe, you will be sure to take the drugs that have been ordered." b. "This therapy is recommended to make sure that you receive the treatment you need and the infection doesn't become resistant to the drugs." c. "This is to make sure you take your medication if your condition becomes so advanced that you do not have enough cerebral oxygenation to remember." d. "Tuberculosis medications are very expensive so this method ensures that government money doesn't get wasted on those who will not take the drugs."

ANS: B It is important to be respectful to clients and to consider their perspective and psychological health while also responding truthfully. Directly observed therapy (DOT) programs for TB medication involve the nurse observing and documenting individual clients taking their TB drugs. When clients prematurely stop taking TB medications, there is a risk that the TB will become resistant to the medications. This can affect an entire community of people who are susceptible to this airborne disease. DOT ensures that TB-infected clients receive adequate medication. Thus, DOT programs are aimed at the population level to prevent antibiotic resistance in the community and to ensure effective treatment at the individual level. Many health departments have DOT home health programs to ensure adequate treatment. The risk to client and the community if the client does not follow the treatment regimen is that the TB will become resistant to these medications. It does not have to do with the possibility of noncompliance by the patient, that the patient will not remember, or related to the cost of the medications.

Which of the following best explains why chlamydia is a major focus of public health efforts? a. It has more serious long-term outcomes than other STDs. b. It can cause problems in infants born to infected mothers. c. It is not frequently seen in the United States. d. It is so difficult and expensive to treat

ANS: B Like gonorrhea and other STDs, chlamydia can cause neonatal complications in infants born to infected mothers. It is treated rather easily with antibiotics. Chlamydia does have serious long-term outcomes for the client, but so do syphilis and other STDs. However, unlike syphilis, which in its later stages is rare in the United States, chlamydia is the most common reportable infectious disease in the United States and hence is a major focus of public health.

A nurse is concerned about the prevalence of tuberculosis among migrant farm workers. Which of the following activities would be best to use when implementing tertiary prevention? a. Administer purified protein derivative (PPD) to contacts of those with tuberculosis. b. Initiate directly observed therapy (DOT) for tuberculosis treatment. c. Provide education about the prevention of tuberculosis to members of the migrant community. d. Use skin tests to screen migrant health workers for tuberculosis infection.

ANS: B Tertiary prevention is carried out among persons already infected with the disease. In this instance, DOT ensures compliance with treatment to cure the disease and to prevent worsening or the development of secondary problems. Administering purified protein derivative (PPD) to contacts of those with tuberculosis is secondary prevention as this is an at-risk population. Providing education about the prevention of tuberculosis to members of the migrant community is a primary prevention strategy as it is aiming to prevent the disease from occurring. Using skin tests to screen migrant health workers for tuberculosis infection is secondary prevention as it is a screening aimed at early detection of the disease.

When a nurse discovers that a woman has been treated for cervical cancer, the nurse asks the woman whether she has ever been tested for HIV or other STDs. The woman is offended and asks why the nurse would ask her such a thing. Which of the following statements would be the best response from the nurse? a. "Cervical cancer treatments may decrease immunity, so that it is easier to acquire STDs." b. "Cervical cancer usually is caused by HPV, and often the presence of one STD is accompanied by other STDs." c. "The presence of an STD in women with cervical cancer may lead to congenital defects in offspring." d. "The presence of an STD in a woman with a history of cervical cancer has been associated with a relapse of the cancer after treatment."

ANS: B The link between HPV infection and cervical cancer has been established and is associated with specific types of the virus. In 80% to 90% of cases of cervical cancer, evidence of HPV has been found in the tumor. Additionally, HSV-2 infection is linked with the development of cervical cancer. Because the presence of an STD increases the risk for the presence of other STDs, it is essential to screen for this information. There is no evidence that the presence of an STD in women with cervical cancer will lead to congenital defects in offspring. The presence of an STD increases the risk for another STD. There is no evidence of relapse of cervical cancer when there is presence of an STD.

A student comes to the college health clinic with typical cold symptoms of fever, sneezing, and coughing, but the nurse also notes small white spots on the inside of the student's cheeks. Which of the following actions should be taken by the college health nurse? a. Inform all students, staff, and faculty of a possible rubella epidemic b. Inform all students, staff, and faculty of a possible measles epidemic c. Reassure the student that it is just a bad cold and will soon pass d. Tell the student to take two acetaminophen and drink lots of fluids

ANS: B - Inform all students, staff, and faculty of a possible measles epidemic Measles is an acute, highly contagious disease that, although considered a childhood illness, is often seen in the United States in adolescents and young adults. Symptoms include fever, sneezing, coughing, conjunctivitis, small white spots on the inside of the cheek (Koplik spots), and a red, blotchy rash beginning several days after the respiratory signs. Measles is serious. Around 10% of measles cases require hospital admission. It can lead to pneumonia and encephalitis, and it can kill. Persons who may have been exposed should be informed that anyone under 18 who has not received both immunization doses should receive measles vaccine.

Which of the following best describes the current goal in relation to communicable diseases? a. To control political borders so diseases cannot spread further b. To exterminate specific infectious agents one by one c. To expand health care facilities to improve infectious disease treatment d. To achieve worldwide immunization to control new cases

ANS: B - To exterminate specific infectious agents one by one The goal of prevention and control programs is to reduce the prevalence of a disease to a level at which it no longer poses a major public health problem. In some cases, diseases may even be eliminated or eradicated. The goal of elimination is to remove a disease from a large geographical area (e.g., a country or region of the world), such as has been done with polio in the Americas. Eradication is the irreversible termination of all transmission of infection by extermination of the infectious agents worldwide, as has been done with smallpox.

Which of the following components of the epidemiologic triangle contributes most to a female client developing a vaginal infection caused by fungi after successful treatment of her strep throat with antibiotics? a. Agent b. Environment c. Host d. Agent and host

ANS: B - environment The antibiotic therapy eliminates a specific pathological agent, but it also may alter the balance of normally occurring organisms in the woman's body, which causes a change in the vaginal environment and allows normally present fungi to proliferate, resulting in a yeast infection.

Which of the following is the number one cause of death worldwide? a. Chronic diseases (heart disease, cancer, stroke) b. Infectious diseases c. Injuries (accidental or purposeful) d. Terrorism

ANS: B - infectious diseases In countries with higher standards of living, where people live longer, chronic diseases—heart disease, cancer, and stroke—are the leading causes of death. Infectious diseases, however, are still the number-one cause of death worldwide.

Which of the following is the most common vector-borne disease worldwide? a. Dengue b. Malaria c. Onchocerciasis (river blindness) d. Yellow fever

ANS: B - malaria Globally, malaria is the most prevalent vector-borne disease, with over 2.4 billion people at risk and more than 275 million cases reported each year. More than 1 million children die of malaria each year. Dengue is the second most common vector-borne disease.

7. A community is examining which programs are needed within the community, the populations they will target, and how they will be funded. Which of the following would be the least risky decision for the community to make? A. Choose whichever option is the least expensive of agency resources B. Choose to do nothing C. Choose whatever the agency administration prefers D. Choose whatever the majority of clients prefer

ANS: B Choose to do nothing The need and demand for a program are determined by working with the client. This stage of planning creates options for solving the problem and considers several solutions. Each option for program solution is examined for its uncertainties (risks) and consequences. A "do nothing" decision is always the decision with the least risk to the provider. DIF: Cognitive Level: Apply (Application) REF: p. 279

16. A committee of health care professionals would like to establish a countywide program to improve Hispanic immigrant access to culturally competent health care services. Which of the following persons would be most helpful as a key informant? A. Hospital administrator B. Hispanic community leader C. National expert on cultural competency D. Politician or county official

ANS: B Hispanic community leader Key informants are leaders in the community who are knowledgeable about community needs. In this scenario, the Hispanic leader most likely knows more about the needs of the Hispanic community than the others listed. DIF: Cognitive Level: Apply (Application) REF: p. 280

12. Evaluation is underway for a statewide program to decrease teen injury and death associated with teens who drive while under the influence of alcohol. Which of the following questions would best be used for the summative evaluation of the program? A. Are program participants continuing to attend the programs, and do their satisfaction scores indicate that they are pleased with the program? B. How do statistics for injuries and deaths associated with drunk driving compare for teens in the year following the program? C. How does the amount of alcohol intake by teens compare before and after participants enter into the program? D. What problems are identified as the program is implemented?

ANS: B How do statistics for injuries and deaths associated with drunk driving compare for teens in the year following the program? Summative evaluation is evaluation to assess program outcomes or as a follow-up of the results of the program activities. Two of the options are examples of questions used for formative evaluation. The goal is addressed in the question about statistics; however, the goal was not to decrease drinking of alcohol but to decrease driving when drinking alcohol. DIF: Cognitive Level: Analyze (Analysis) REF: p. 277

When writing to a legislator to advocate for funds to support advanced practice nursing education, a nurse should remember to (select all that apply): a. Limit the letter to one page in length. b. Identify oneself as a nurse. c. Provide only factual information. d. Share personal knowledge about the issue.

ANS: B, D When writing to a legislator, the letter should be a maximum of two pages in length and opinions should be expressed. Additionally, the nurse should identify oneself as a nurse and share knowledge about the issue.

4. Which would be an appropriate descriptor that meets all criteria for defining a client to be served by a program? A. All women ages 40 to 50 who have not had a menstrual period for 3 consecutive months B. Immigrants residing in Central County for less than 5 years who have difficulty understanding care instructions because of limited English proficiency C. Pregnant women who have received nutritional counseling but whose nutritional status did not improve D. Children ages 18 months to 5 years old who have been treated for nutritional deficiencies at the Central County Clinic

ANS: B Immigrants residing in Central County for less than 5 years who have difficulty understanding care instructions because of limited English proficiency The client should be defined by biological and psychosocial characteristics, by geographical location, and by the problems to be addressed. For example, in a community with a large number of preschool children who require immunizations to enter school, the client population may be described as all children between 4 and 6 years of age residing in Central County who have not had up-to-date immunizations. This example tells the reader who the client is, what the need is, how large the population is, and where they are located. DIF: Cognitive Level: Analyze (Analysis) REF: pp. 277-278

19. Which of the following programs demonstrates the use of tertiary prevention? A. Developing an in-school clinic that provides birth control counseling and contraception B. Providing a diabetes management program for persons with diabetes mellitus C. Providing cardiovascular fitness evaluations at annual health fairs D. Setting up free blood pressure screenings at popular department stores and supermarkets

ANS: B Providing a diabetes management program for persons with diabetes mellitus The aim of tertiary prevention programs is to reduce complications from disease. Developing an in-school clinic is a primary prevention (pregnancy has not occurred). Fitness evaluations at health fairs and blood pressure screenings are secondary prevention programs (screening identifies conditions early and determines incidence/prevalence). DIF: Cognitive Level: Analyze (Analysis) REF: p. 284

3. Which of the following best describes the purpose of strategic planning? A. To anticipate client needs now and in the future B. To match client needs, provider strengths, and agency resources C. To maximize effective use of agency resources D. To utilize provider strengths and competencies

ANS: B To match client needs, provider strengths, and agency resources Strategic planning involves matching client needs, provider strengths and competencies, and agency resources. Everyone involved can anticipate what will be needed to implement the program, what will occur during implementation, and what the outcomes will be. DIF: Cognitive Level: Understand (Comprehension) REF: p. 277

The public health nurse comes to the hospital to see a client just diagnosed with hepatitis A. The nurse says, "I'm sorry to bother you when you're not feeling well, but I need to ask you a few questions." Which of the following questions would be most appropriate for the nurse to ask the client? (Select all that apply.) a. "Do you know how you got this infection?" b. "Who lives with you?" c. "Where are you employed?" d. "Where do you usually eat?" e. "Are you sexually active?"

ANS: B, C, E The nurse should ask about sexual contacts and ask who (if anyone) lives with the client because the named individuals will need to have immune globulin administered to hopefully prevent the spread of hepatitis A and a community epidemic. The nurse should also ask about the client's place of employment because certain settings warrant special considerations. For example, in restaurants, hospitals, daycare centers, or other institutions, the lack of careful hand-washing by an infected worker can result in contamination of many others. Hepatitis A can be spread through food contaminated by an infected food-handler, contaminated produce, or contaminated water. However, this mode of transmission is not very common in the United States.

The advanced practice nurse explains that the client has an upper respiratory infection (URI) and suggests several measures that might make the client more comfortable. Which of the following best describes why the nurse doesn't just prescribe antibiotics as the client repeatedly requests? (Select all that apply.) a. Antibiotics are expensive, whereas the support measures would be almost free of cost. b. Viral diseases are not affected by antibiotics. c. Clinics cannot afford to continually give antibiotics to anyone who asks for them. d. The more antibiotics are prescribed, the more infectious agents develop resistance to such drugs.

ANS: B, D Antibiotics are not effective against viral diseases, a fact found unacceptable to many clients looking for relief from the misery of a cold or flu. The inappropriate prescribing of antibiotics contributes to the growing problem of infectious agents that have developed resistance to once-powerful antibiotics.

Six students order meals at a local restaurant. Which of the following students are at highest risk for illness? (Select all that apply.) a. The first student asks for a salad with chicken strips and dressing on the side. b. The second student asks for a hamburger, very rare. c. The third student orders a tuna salad sandwich with extra mayonnaise. d. The fourth student orders a breakfast meal with two very soft-poached eggs and toast.

ANS: B, D Escherichia coli O157:H7 can produce a strong cytotoxin that can cause a potentially fatal hemorrhagic colitis. Hamburger is often involved in outbreaks, since the grinding process exposes pathogens on the surface of the whole meat to the interior of the ground meat, effectively mixing the once-exterior bacteria thoroughly throughout the hamburger so that searing the surface no longer suffices to kill all bacteria. Also, hamburger is often made of meat ground from several sources. The best protection against foodborne pathogens is to thoroughly cook food before eating it. Salmonella is also transmitted by undercooked foods such as eggs, poultry, dairy products, and seafood. Consequently, students eating very rare hamburger and undercooked eggs are at high risk.

A nurse will be using an interpreter during a client encounter. What considerations should be made by the nurse? Select all that apply. a. It is appropriate to use family members as interpreters. b. Written materials should be available in the clients primary language. c. Observe the interpreters gestures to assure client understanding. d. The gender, age, and educational level of the interpreter should be evaluated.

ANS: B, D Family members should be used with caution. The clients gestures and non-verbal messages should be observed to assure understanding. Written materials should be available in the clients primary language. The gender, age, educational level, socioeconomic status, religion, and dialect should all be considered when selecting the proper interpreter.

The most chronic bloodborne infection in the United States is: a. Hepatitis A b. Hepatitis B c. Hepatitis C d. HIV

ANS: C The most chronic bloodborne infection in the United States is hepatitis C.

In which of the following cases would the school nurse be correct to advise the parents of an HIV-infected child to keep the child home from school? (Select all that apply.) a. The child develops allergies with sneezing. b. The child is unable to control body secretions. c. The nurse is not comfortable with being responsible for the child. d. There is an outbreak of chickenpox in the school. e. The child persists in biting behavior.

ANS: B, D, E Not attending school may be advisable if cases of childhood infections, such as chickenpox or measles, occur in the school, because the immunosuppressed child is at greater risk for suffering complications. Alternative arrangements, such as homebound instruction, might be instituted if a child is unable to control body secretions or displays biting behavior. HIV-positive children are encouraged to obtain routine immunizations, because their immune systems are compromised and they are more susceptible to such infections. To date, no cases of HIV infections being transmitted in a school setting have occurred in the United States. Thus, the nurse should not be fearful of providing care to this child.

A nurse is completing a health risk appraisal with a client. Which of the following comments would cause the nurse to probe further to determine if the family is in crisis? a."I can't visit my husband in the hospital when I'm at work all day. How can I be sure he's all right?" b."My husband always handled our finances. Now that he's gone, I'll have to learn how to do this." c."I don't know what to do now that my husband is dead. There is no way I can go back to work and also take care of our three children." d."What am I supposed to do now that everything we own is gone? Are there any agencies that can help me?"

ANS: C A family crisis occurs when the family is not able to cope with an event and becomes disorganized or dysfunctional. When the demands of the situation exceed the resources of the family, a family crisis exists. In three of the options, the survivor is considering the problem and trying to learn how to cope or seek resources to cope. Only in this option is the person overwhelmed and unable to conceive of how to cope

Which of the following families is at high risk for health problems? a.A man agrees that he needs to eat better and exercise more but also expresses how busy he is at his job. b.A man knows that his grandfather, father, and older brother all died of cardiac disease. c.A man is currently unemployed and despairs about finding a position. d.A man expresses disappointment that, having been laid off as an executive, his new position pays only about two thirds of his original salary

ANS: C A person who is unemployed and despairing of finding employment is at serious economic risk, which is one of the foremost predictors of health problems. Economic risk is determined by the relationship between family financial resources and the demands on those resources. Having adequate financial resources means that a family is able to purchase the necessary services and goods related to health, such as adequate housing, clothing, food, education, and health or illness care.

A person has syphilis with signs and symptoms of rash, sore throat, and muscle and joint pain. This person is experiencing which stage of syphilis? a. Congenital b. Primary c. Secondary d. Tertiary

ANS: C A person with signs and symptoms of a rash, sore throat, and muscle/joint pain is experiencing the secondary stage of syphilis.

Nurses working in hospice and home care settings need to be knowledgeable of health policy to: a. Provide safe nursing care. b. Prevent elder abuse. c. Receive payment for the services provided. d. Influence future legislation.

ANS: C Although nurses should be involved in influencing future legislation by contacting their legislators, the best response is that nurses need to be knowledgeable of the current health policies needed so that payment can be received for services that are providing.

A nurse wants to obtain information on the alternative methods of health care her 45-year-old female client uses. Who is the best person to ask about this? a. The husband of the client b. A community leader of the ethnic group c. The client herself d. The religious leader of the ethnic group

ANS: C Clients provide a rich source of information about their own cultures.

The tendency to ignore all differences between cultures and to act as though the differences do not exist is defined as cultural: a. Conflict b. Shock c. Blindness d. Imposition

ANS: C Cultural blindness occurs when people state that everyone is treated the same, regardless of their cultural orientation.

When a nurse supports the use of traditional home remedies in conjunction with Western medicine to promote healthy behaviors, the nurse is using cultural: a. Accommodation b. Brokering c. Preservation d. Repatterning

ANS: C Cultural preservation refers to assistive, supportive, facilitative, or enabling nurse actions and decisions that help the clients of a particular culture to retain and preserve traditional values, so they can maintain, promote, and restore health.

A nurse resigns from a position in a hospital to accept a job in a community setting. After starting the new job, the nurse feels helpless and confused, wondering if this was the right career choice. This nurse is likely experiencing cultural: a. Conflict b. Relativism c. Shock d. Brokering

ANS: C Culture shock can happen to individuals within their own culture when they are having experiences such as starting a new job or career. Culture shock is brought on by anxiety from losing familiar signs and symbols of social interaction. Feelings associated with culture shock are helplessness, discomfort, and disorientation.

While researching various health care services that are provided in prisons, a nursing student learns that these services: a. Are funded by the Department of Defense b. Focus on health promotion interventions c. Must be provided at an adequate level d. Are available to those diagnosed with a chronic illness

ANS: C Health care services for prisoners are required at an adequate level for all incarcerated individuals. This is administered by the Department of Justice. Prisoners will receive health care services when diagnosed with chronic illnesses as well as acute illnesses.

An example of a bacterial STD is: a. Trichomonas b. HIV c. Syphilis d. Genital warts

ANS: C Trichomonas, HIV, and genital warts are caused by viruses.

A PHN employed by the state's department of health is working on a team to develop local health policy. The nurse recognizes that policy development focuses on the level of the larger society and adopts strategies that require political actions such as lobbying and testifying. The reason that action in the policy arena comes most easily and naturally to nurses is the fact that the policy process is very similar to which of the following? A. Citizen action committee B. Nursing diagnosis C. Nursing process D. Socratic method

ANS: C Health policy is simply the process of turning health problems into workable action solutions. Thus, the policy process is very similar to the nursing process, but the focus is on the level of the larger society and the adoption of these strategies requires political action. The policy process includes statement of a health care problem, statement of the policy options to address the problem, adoption of a particular policy option, implementation of the policy product, and evaluation of the policy's intended and unintended consequences in solving the original health problem.

Which statement about race is true? a. In the United States, children of biracial parents are usually assigned the race of the father. b. Ethnicity and race are synonymous terms. c. Individuals may be of the same race but of different cultures. d. No social significance is usually placed on race.

ANS: C It is often a misconception that persons of the same race have the same culture. For example, African Americans may have been born in Africa, the Caribbean, North America, or elsewhere and have very different cultures.

When a nurse is sued for malpractice and goes to court, which type of law is being used? a. Constitutional b. Common c. Judicial d. Institutional

ANS: C Judicial law is based on court or jury decisions.

Which of the following best describes a normative life event that can increase the risk for illness? a.A family is involved in a motor vehicle crash. b.A group of teens experiment with recreational drugs. c.A woman is pregnant with her first child. d.The family wage earner is laid off from his job

ANS: C Life events can increase the risk for illness and disability. Normative events are those that are generally expected to occur at a particular stage of development or of the life span. Although pregnancy is a normal condition, it carries risks such as the development of eclampsia or more minor health problems such as constipation and hemorrhoids. Additionally,pregnancy (and the birth that follows) will alter family dynamics and may increase risk for psychological stressors. The events listed in the other options are not normative life events

A nurse is drawing a genogram. Which of the following would the nurse use to demonstrate a marriage relationship between two individuals? a.A broken line b.A dashed line c.A solid line d.Two parallel lines

ANS: C Marriage is indicated by a solid line on a genogram

When working with a client who is HIV positive, the nurse serves as an educator, teaching about the modes of transmission, and serves as a(n): a. Advocate, lobbying for AIDS research b. Counselor, discussing implications of future sexual activity c. Role model, providing supportive care d. Policy maker, addressing laws governing privacy rights of HIV-positive persons

ANS: C Nurses must be role models because many HIV-positive patients are stigmatized.

Which of the following clients would cause the nurse the most concern? a.The client who is currently unemployed but actively seeking a position and frequently walking from one interview to another b.The client who is not employed but spending time at the gym keeping fit and studying the benefits of organic natural uncooked foods c.The client who is employed and often works 12 hours a day without moving from the computer desk d.The client who is employed but always leaves promptly at 5:00 to pick up the children from the day care center

ANS: C Personal health habits continue to contribute to the major causes of morbidity and mortality. The pattern of personal health habits and behavioral risk defines individual and family lifestyle risk. The client who doesn't move from the computer desk is creating great stress andstrain on personal physiology and needs to be educated on the benefits of exercise and the risks of cumulative trauma on the body. Multiple health benefits of regular physical activity have been identified; regular physical exercise is effective in promoting and maintaining health and preventing disease

Using the Neuman Systems Model, which of the following questions would the nurse ask a client to assess physiological health? a."What helps you to cope with situations involving your wife's cancer?" b."How has your child's illness affected the behavior of your other children?" c."Tell me about any illnesses your other family members have." d."Who do you turn to for support outside your immediate family?

ANS: C Physiological health involves issues related to physical wellness or illness. Other components of health in Neuman's model include psychological health, sociocultural health, developmental health, and spiritual health.

A community health nurse visits a homeless shelter that has several clients with tuberculosis who are taking antimicrobial drugs. Supervised drug administration may be done on a daily basis with persons with poor adherence because: a. This type of client does not care about his or her health. b. Homeless people have no access to medications. c. Poor adherence can result in drug resistance. d. The antimicrobials are so powerful, clients must be observed for reactions.

ANS: C Poor adherence has lead to antibiotic resistant strains.

A nurse implements nursing interventions considering the uniqueness of the persons culture. The practice is called cultural: a. Diversity b. Knowledge c. Competence d. Awareness

ANS: C Providing care based on the uniqueness of the clients cultural norms and values is one of the four guiding principles of culturally competent nursing care.

A persons skin color is an example of: a. Multiculturalism b. Ethnicity c. Race d. Culture

ANS: C Race is primarily a social classification that relies on physical markers.

Segregation is an example of: a. Prejudice b. Cultural imposition c. Racism d. Stereotyping

ANS: C Racism is a form of prejudice that occurs through the exercise of power by individuals and institutions against people who are judged to be inferior in, for example, intelligence, morals, beauty, and self-worth.

A 35-year-old man from Russia comes to the United States seeking asylum because of religious persecution in his native country. This type of immigrant is known as a(n): a. Legal immigrant b. Lawful permanent resident c. Refugee d. Unauthorized immigrant

ANS: C Refugees are people who seek protection in the United States because of fear of persecution in their homeland.

A nurse is implementing risk reduction interventions with a family. Which of the following questions is most important for the nurse to ask? a."Did any of the hunters in your family kill a deer this year?" b."How do you keep your rifles safe from curious children?" c."Where do you shoot with your handguns?" d."Where do you keep your rifles locked when it is not hunting season?"

ANS: C Risk reduction is a complex process that requires knowledge of risks and families' perceptionsof the nature of the risk. In this situation the nurse was asking questions to determine the family's perception of risks associated with owning guns. If the family does not perceive the behavior (having guns in the house) as risky, but rather as necessary for food or sport, the nurse must first educate or persuade the family that others may be more comfortable if certain precautionary measures are taken (such as locks on the guns). Rifles are used for food and sport but handguns are often used in crimes and accidents resulting in death

The practices of physicians, nurses, and other health care providers are differentiated by the: a. Health Resource and Services Administration (HRSA) b. State board of nursing c. Scope of practice d. United States Department of Health and Human Services

ANS: C Scope of practice involves defining nursing, setting its credentials, and then distinguishing between the practices of nurses, physicians, and other health care providers.

Hispanics tend to believe that the needs of the family take priority over those of the individual. This is an example of cultural variations of: a. Communication b. Space c. Social organization d. Environmental control

ANS: C Social organization refers to the way in which a cultural group structures itself around the family to carry out role functions.

Nursing practice is governed by: A. boards of nursing established by the U.S. Department of Health and Human Services. B. nurse practice acts promulgated by federal and state legislative boards. C. nurse practice acts promulgated by state legislatures and operationalized by the state boards of nursing. D. nurse practice acts promulgated by the states in conformity with broad federal guidelines.

ANS: C State legislatures enact laws that establish boards of nursing. The functions of the board are described by the nurse practice act of each state. The boards of nursing license nurses, oversee training programs, and interpret and enforce statutory law.

The nurse teaches the family of an AIDS client about managing symptomatic illness by preventing deteriorating conditions, such as diarrhea, skin breakdown, and inadequate nutrition. This nursing intervention is an example of _____ prevention. a. Primary b. Secondary c. Tertiary d. Primary health care

ANS: C Tertiary prevention includes those interventions aimed at disability limitation and rehabilitation from disease, injury, or disability.

A nurse is interested in census data. Which federal departments website would be a good place to start? a. Department of Labor b. United States Immunization Survey c. Commerce Department d. World Health Organization

ANS: C The Commerce Department handles the United States Census and population estimates and projections.

The Office of Homeland Security has a mission to rebuild the crumbling public health infrastructures of each state and provide: a. Vaccines for smallpox, anthrax, and plague b. Regulation of nuclear weapons c. Surveillance of bioterrorism threats d. Regulation of foreign nurses entering the United States

ANS: C The DHHS and the new federal agency, the Office of Homeland Security, have provided funds to address serious bioterrorism threats to the people of the United States.

A nurse seeks the best resource that provides a broad vision of the goals and objectives of many health care stakeholders in our nation and identifies the current national health policy for the United States. The best source for such information would be the: A. American Public Health Association's guidelines. B. website of the Centers for Disease Control and Prevention. C. Healthy People initiatives. D. Pan American Health Organization's mission statement.

ANS: C The Healthy People initiatives began in 1979 with subsequent updates and revisions across multiple decades. Healthy People 2020 builds on earlier versions to identify a national health agenda to attain quality, longer lives free of preventable diseases, disability, injury, and premature death; achieving health equity, eliminating health disparities and improving health for all groups; creating social environments that promote good health for all; and promoting quality of life, healthy development, and healthy behaviors across all life stages.

The first major federal government action relating to health was the: a. Passage of the Social Security Act b. Creation of the National Institutes of Health (NIH) c. Creation of the Public Health Service (PHS) d. Creation of Medicare and Medicaid

ANS: C The PHS was created in 1798.

A nurse demonstrates the desire to provide culturally competent care by: a. Relying on a textbook for information about an ethnic group b. Bringing a translator to the local community clinic c. Taking Spanish classes in the evening at a local college d. Judging others using his or her own cultural values

ANS: C The activity that suits the definition of cultural desire is one that a nurse would want to do instead of being directed to do so, referring to the intrinsic motivation of the nurse.

A nurse is found to be negligent. Because of the doctrine of respondeat superior, the one responsible for that negligence is the: a. Administrator b. Nurses immediate supervisor c. Nurses employer d. Nurse

ANS: C The doctrine of respondeat superior says that when a nurse is employed and functioning within the scope of that job, the one responsible for that negligence is the nurses employer.

A nurse would like to propose legislation requiring all schools to employ a registered nurse. The first step taken in the legislative process would be: a. Introducing the bill to the legislature b. Moving the bill to the other house c. Developing the policy option into a bill d. Hearings, testimony, and lobbying

ANS: C The first step in the legislative process is developing the policy option into a bill. This is then followed by introducing the bill to the legislature; hearings, testimony, and lobbying; and the bill moving to the other house.

Which branch of the government interprets the states rights to grant abortions? a. Executive branch b. Legislative branch c. Judicial branch d. Federal branch

ANS: C The judicial branch interprets laws and their meaning, as in its ongoing interpretation of states rights to define access to reproductive health services for citizens of the states.

In the United States, which group is most affected by a new HIV diagnosis? a. Women having heterosexual relationships b. White IV drug abusers c. Young Black gay and bisexual men d. Men who have sex with other men

ANS: C Although new HIV diagnosis in general has declined, young Black gay and bisexual men are the most affected, with an 94% increase in diagnosis. The largest number of new infections in 2018 was in men who had sex with other men, and this was followed by heterosexual transmission. HIV infections in women are primarily due to heterosexual contact or IV drug use. This information supports the incorrectness of the other options.

Which of the following sexually transmitted diseases can be prevented through immunization? a. Chlamydia b. Gonorrhea c. Hepatitis B d. Genital Herpes Simplex

ANS: C Hepatitis B infection can be prevented by immunization; vaccines for the remaining options are not yet available. There are not yet vaccines for chlamydia, gonorrhea, or genital herpes simplex.

A nurse is providing education to a client about the use of PrEP. Which of the following statements would the nurse include as part of this teaching? a. "Side effects of PrEP include extreme lethargy and joint pain." b. "PrEP has been shown to be effective in preventing transmission of the disease from sharing needles." c. "The effectiveness of PrEP will depend on your adherence to the medication regimen." d. "PrEP will prevent you from contracting HIV and hepatitis B."

ANS: C Pre-exposure prophylaxis, or PrEP, is a new HIV prevention method for people who do not have the infection but would like to reduce their risk of becoming infected. PrEP requires taking a pill to prevent the HIV virus from getting into the body. This prevention method requires strict adherence to taking the medication and having regular HIV testing; it is also used in combination with other HIV prevention methods rather than in isolation (CDC: PrEP, 2012b). It has been shown to be effective for people at very high risk for HIV infection through sex; the results about its effectiveness with injection drug users are not yet available. Extreme lethargy and joint pain are not common side effects of PrEP. PrEp provides protection against HIV, not against hepatitis B.

Which of the following provides the best explanation as to why people do not immediately seek medical treatment when they first become ill with HIV? a. They are afraid to get tested for fear that the results will be positive. b. They avoid the problem (maybe it will go away). c. They don't recognize their symptoms as possibly being due to HIV. d. It is too expensive to get an HIV test.

ANS: C When HIV enters the body, it can cause a flu-like syndrome referred to as a primary infection or acute retroviral syndrome. This may go unrecognized. The symptoms are similar to flu or a bad cold including sore throat, lethargy, rash, fever, and muscle pain. An antibody test at this stage is usually negative. So the person or a medical provider may not recognize the illness as HIV. Thus, it is not worthwhile to be tested during this stage of the illness. This information supports the incorrectness of the other options

Which of the following public health actions has been particularly instrumental in reducing childhood infectious diseases in the United States? a. Answering parents' questions about the safety and importance of vaccines today b. Educational campaigns to all health care providers about the importance of immunizations whenever a child is seen c. "No shots, no school" legislation, which legally requires children be immunized before school d. Offering all immunizations to all children free of any charge

ANS: C - "No shots, no school" legislation, which legally requires children be immunized before school Vaccines are one of the most effective methods of preventing and controlling communicable diseases. Hopefully, all nurses answer questions, remind colleagues to think about immunizations whenever a child is seen, and encourage continuing free or low-cost immunization clinics. One of the most effective programs has been the "no shots, no school" legislation, which has resulted in the immunization of most children by the time they enter school.

A man loudly protests his increased property tax bill right after the public health department has made a plea for more funds. "Why," he asks, "should my tax dollars be used to pay for their children to be immunized?" Which of the following would be the best response by the nurse? a. "Immunizations are required by law, and if their parents can't afford it, you and I will have to pay for it." b. "It's just the right thing to do." c. "Only by making sure most kids are immunized can we stop epidemics that might hurt all of us." d. "We're a religious God-fearing community, and we take care of each other."

ANS: C - "Only by making sure most kids are immunized can we stop epidemics that might hurt all of us." Herd immunity is the resistance of a group of people to invasion and spread of an infectious agent because a high proportion of individual members of a group are resistant to the infection. Higher immunization coverage will lead to greater herd immunity, which in turn will block the further spread of the disease.

An instructor is reviewing Salmonella infections with her class. Which of the following comments indicates that the student needs further review on how Salmonella is spread? a. "Certain pets and farm animals may be Salmonella carriers." b. "It is possible to transmit Salmonella by person-to-person contact." c. "Salmonella may be spread by spores that form once contaminated blood is exposed to the air." d. "Salmonella outbreaks are usually due to contaminated meat, poultry, and eggs."

ANS: C - "Salmonella may be spread by spores that form once contaminated blood is exposed to the air." Meat, poultry, and eggs are the foods most often associated with salmonellosis outbreaks. Animals are the common reservoir for the various Salmonella serotypes, although infected humans may also fill this role. Animals are more likely to be chronic carriers. Reptiles such as iguanas have been implicated as Salmonella carriers, along with pet turtles, poultry, cattle, swine, rodents, dogs, and cats. Person-to-person transmission is an important consideration in daycare and institutional settings. Anthrax (not Salmonella) forms spores when infected blood is exposed to air.

Asians may perceive illness as disharmony with other forces and that medicine is only able to relieve the symptoms rather than cure the disease. They may look to naturalistic solutions and acupuncture to resolve or cure health problems. This is an example of cultural variations of: a. Communication b. Space c. Social organization d. Environmental control

ANS: D Environmental control refers to the ability of individuals to control nature and to influence factors in the environment that affect them.

A nurse believes additional funding is necessary to support a home-delivered meal program for the elderly. The best way to share these ideas with a legislator is to: a. Send an e-mail. b. Write a formal letter. c. Attend a town hall meeting. d. Schedule a face-to-face visit.

ANS: D Face-to-face visits with legislators are viewed as the most effective means of communication.

Which of the following data would most likely be collected in a syndromic surveillance system? a. Incidence of bioterrorism attacks b. Number of air travelers c. Incidence of school absenteeism d. Number of influenza vaccines administered

ANS: C - Incidence of school absenteeism Syndromic surveillance systems use existing health data in real time to provide immediate analysis and feedback to those charged with investigation and follow-up of potential outbreaks. These systems incorporate factors such as the previously mentioned temporal and geographic clustering and unusual age distributions with groups of disease symptoms or syndromes (e.g., flaccid paralysis, respiratory signs, skin rashes, gastrointestinal symptoms) with the goal of detecting early signs of diseases that could result from a bioterrorism-related attack. Syndromic surveillance systems may include tracking emergency department visits sorted by syndrome symptoms as well as other indicators of illness including school absenteeism and sales of selected over-the-counter medications. In recent years, the tracking of cold medicines used to make crystal methamphetamine has received considerable attention.

A community is experiencing an epidemic of the measles. The nurse is trying to determine if this problem is happening in other communities as well. Which of the following resources should the nurse use to answer this question? a. Centers for Disease Control and Prevention Weekly Report b. Communicable Diseases Weekly Report c. Morbidity and Mortality Weekly Report d. Weekly National Report of Communicable Diseases

ANS: C - Morbidity and Mortality Weekly Report Requirements for disease reporting in the United States are mandated by state rather than federal law. The list of reportable diseases varies by state. State health departments, on a voluntary basis, report cases of selected diseases to the Centers for Disease Control and Prevention (CDC) in Atlanta, Georgia. The National Notifiable Diseases Surveillance System (NNDSS) data are collated and published weekly in the Morbidity and Mortality Weekly Report (MMWR).

Which of the following is the most common vector-borne disease in the United States? a. Babesiosis b. Ehrlichiosis c. Lyme disease d. Rocky Mountain spotted fever

ANS: C - lyme disease All four are diseases borne by ticks as the vectors. Lyme disease became a nationally notifiable disease in 1991 and is now the most common vector-borne disease in the United States.

2. Which of the following best describes the ultimate goal of program planning? A. Avoid unanticipated conflicts in the program development phase B. Provide adequate funding to meet the program's resource requirements C. Ensure that health care services are acceptable, equal, effective, and efficient D. Prevent unnecessary duplication of services

ANS: C Ensure that health care services are acceptable, equal, effective, and efficient Although the other options are aspects of program planning, they address only limited concerns. The comprehensive goal of program planning is to ensure that health care services are acceptable, equal, efficient, and effective. DIF: Cognitive Level: Understand (Comprehension) REF: p. 277

8. A nurse checks health department records to compare the number of new teen clients presenting for birth control counseling and management in the 2 months before and after an education intervention program to decrease teen pregnancy. Which of the following steps of the evaluation process is being completed by the nurse? A. Engage stakeholders B. Justify conclusions C. Gather credible evidence D. Focus on the evaluation design

ANS: C Gather credible evidence When the nurse gathers credible evidence, the following information is collected: indicators that will be used, sources of data, quality of the data, quantity of information to be gathered, and the logistics of the data gathering phase. Data gathered should provide credible evidence and should convey a well-rounded view of the program. DIF: Cognitive Level: Apply (Application) REF: p. 283

11. A nurse is completing a summative evaluation of a program designed to decrease obesity in school-age children. Which of the following is the most important question for the nurse to ask? A. Are school-age children satisfied with the program? B. Can parents and guardians support the program requirements? C. Has obesity in school-age children decreased? D. What is the program cost compared with the program benefit?

ANS: C Has obesity in school-age children decreased? Summative evaluation looks at the end result of the program. The major benefit of program evaluation is that it shows whether the program is meeting its purpose. It should answer the following questions: Are the needs for which the program was designed being met? Are the problems it was designed to solve being solved? If the program does not achieve the purpose for which it is designed, important concerns of satisfaction and cost are irrelevant. So if the program purpose is to decrease obesity, the outcome of importance is a decrease in obesity. DIF: Cognitive Level: Apply (Application) REF: p. 277

14. Based on projected increases in the number of older U.S. citizens, a planning committee wants to establish a day care program for the community's older adult population. During which stage is the need for this program being assessed? A. Inactive stage B. Interactive stage C. Preactive stage D. Reactive stage

ANS: C Preactive stage The preactive stage is one in which assessment is based on the projection of a future need. DIF: Cognitive Level: Apply (Application) REF: p. 278

Which of the following symptoms suggests smallpox as opposed to the more common and much less dangerous chickenpox? (Select all that apply.) a. Child appears only mildly ill until late stages in smallpox. b. Lesions appear in various stages in the same area of the body rather than all at once. c. Rash lesions are most abundant on the face and extremities, not on the trunk. d. Rash occurs 2 to 4 days after sudden onset of fever rather than with the fever.

ANS: C, D Symptoms of smallpox include rash lesions on face and extremities, rash that occurs 2 to 4 days after onset of fever, and vesicles that are deep seated.

A nurse wants to empower the family of a mother who has been newly diagnosed with breast cancer. Which of the following actions would the nurse most likely take?a.Apply for emergency financial assistance on the family's behalf. b.Arrange for community members to assist with child care. c.Invite the mother to join a cancer support group. d.Teach the family how to navigate the health care system

ANS: D Definitions of empowerment reflect three characteristics of the empowered family seeking help: access and control over needed resources, decision-making and problem-solving abilities, and the ability to communicate and to obtain needed resources. Approaches for helping individuals and families assume an active role in their health care should focus on empowering, rather than giving direct help

A nurse is conducting a health risk appraisal. Which of the following activities is the nurse assessing when using this tool? a.Health promotion activities b.Illness prevention activities c.Risk reduction activities d.Unhealthy activities

ANS: D Health risk appraisal refers to the process of assessing for the presence of specific factors in each of the categories that have been identified as being associated with an increased likelihood of an illness, such as cancer, or an unhealthy event, such as an automobile accident.Therefore, the nurse would assess for unhealthy behavior and activities in the areas of biological and age-related risk, social and physical environment risk, and behavioral risk. Activities to promote health, prevent illness, or reduce risks would be advantageous, not risky

A nurse is working in a public health center. A patient who has been newly diagnosed as HIV positive comes for counseling. By law, what must the nurse do? a. Give antiviral medications to the patient. b. Ask the person to name all of his or her sexual contacts. c. Refer the patient to the social worker as someone possibly needing case management. d. Report the HIV-infected person to the state health department.

ANS: D It is mandatory to report HIV cases in all states. The information is then sent to the Centers for Disease Control and Prevention (CDC). Some states report HIV infections either by name or by code.

An example of a cultural encounter is: a. Sharing significant assessment findings with members of a racial minority b. Visiting the native land of the clients served at a community health center c. Telephoning the priest at a Latino church to discuss the health issues of a client d. Learning about traditional healing practices from an American Indian client

ANS: D Learning about traditional healing practices is an example of a direct cultural encounter. This occurs when a nurse engages in cross-cultural interactions.

A goal of the Division of Nursing is to: a. Enhance nursing competence in providing high-tech skills b. Apply disease prevention, environmental health, and health promotion concepts c. Provide scholarships for advanced nursing education d. Enhance racial and ethnic diversity in the nursing workforce

ANS: D One of the Division of Nursing goals is to enhance racial and ethnic diversity and cultural competency in the nursing workforce. The other statements apply to other agencies or are false.

Action taken to protect the health, safety, and welfare of citizens, such as requiring immunizations of children before admission to school, is called what kind of power? a. Executive b. Legislative c. Judicial d. Police

ANS: D Police power is state power concerning health care. This power allows states to act to protect the health, safety, and welfare of their citizens.

The case rate of tuberculosis (TB) is highest among which ethnicity in the United States? a. African American b. Native American c. Hispanic d. Asian

ANS: D TB is most often found in Asian Americans, at 25.6 new TB cases per 100,000 people.

The federal government provides for the protection of the publics health by: a. Interpreting decisions related to womens right to privacy b. Setting up immunization clinics for smallpox vaccine c. Regulating nursing at the state level d. Monitoring the influx of disease at the borders

ANS: D The United States Constitution gives the federal government the authority to provide for the protection of the publics health. By monitoring the influx of disease at the borders the publics health is being protected.

The most common reportable infectious disease in the United States is: a. Gonorrhea b. Syphilis c. Herpes d. Chlamydia

ANS: D The most common reportable infectious disease in the United States is Chlamydia.

A client comes into the clinic and tells the nurse he goes to an acupuncturist for pain. The nurse says he should take pain medication instead. This would be called cultural: a. Conflict b. Blindness c. Relativism d. Imposition

ANS: D This is an example of cultural imposition, the act of imposing ones cultural beliefs, values, and practices on individuals from another culture.

In the Vietnamese culture, individuals may focus on wishes and memories of their ancestors and look to them to provide direction for current situations. This is an example of cultural variations of: a. Communication b. Space c. Social organization d. Time

ANS: D This is the duration or period between successive events, where some cultures assign greater or lesser emphasis to events that occur in the past, present, or future.

Which group should receive an injection of prophylactic immune globulin for possible exposure to hepatitis A? a. Persons who have had direct contact with blood b. Those who ate at the same restaurant as the person with hepatitis A c. All health care workers d. All those who had household or sexual contact with persons with hepatitis A

ANS: D Those who have been in close contact with persons who develop hepatitis A should receive immune globulin.

A student asks the nurse at the student health clinic how AIDS is diagnosed. Which of the following statements would be the best response by the nurse? a. "A diagnosis of AIDS is made when a screening test called an enzyme-linked immunosorbent assay (ELISA) is confirmed by the Western blot test." b. "A diagnosis of AIDS is made when antibodies to HIV are detected about 6 weeks to 3 months following possible exposure." c. "A diagnosis of AIDS is made when antibodies to HIV reach peak levels of 1000/ml of blood." d. "A diagnosis of AIDS is made when CD4 T-lymphocytes drop to less than 22 cells/mm."

ANS: D AIDS is defined as a disabling or life-threatening illness caused by HIV; it is diagnosed in a person with a CD4 T-lymphocyte count of less than 22 cells/ml with or without documented HIV infection. The HIV antibody test (usually the EIA) is the most commonly used screening test for determining whether the antibody to HIV is present but does not confirm AIDS. Positive results with the EIA are tested further with the Western blot test. However, false-negative results are frequent between 6 weeks and 3 months following exposure.

There is great concern in the nurse's community over three local cases of West Nile virus. Which of the following actions should the nurse take to get the community involved in addressing this problem? a. Ask the state department of health for assistance b. Demand that everyone over age 65 become immunized immediately c. Encourage immunization of all children under 12 d. Have an educational campaign to remove any containers of standing water

ANS: D - Have an educational campaign to remove any containers of standing water Periodic outbreaks of West Nile virus appear to result from a complex interaction of multiple factors, including weather—especially hot, dry summers followed by rain, which influences mosquito breeding sites and population growth. Removing standing water will remove mosquito breeding sites.

A nurse was reading PPD tests 24 hours after another nurse had administered them. Which of the following findings would cause the nurse to interpret the test as positive? a. 15 mm of erythema in a client with HIV infection b. 5 mm of induration in an immigrant from a country where TB is endemic c. A 5-mm ruptured pustule with purulent drainage in a homeless client d. 10 mm of swelling and increased firmness in a client recently released from a correctional facility

ANS: D For a PPD test to be positive, induration (swelling with increased firmness) must be present. A diameter of 10-mm induration would be a positive finding in an immigrant from a region with high TB infection. Erythema alone does not indicate a positive finding. A 5-mm induration is not large enough to indicate a positive finding. A small pustule in a homeless client undoubtedly is an infection but may not be due to the PPD test.

The correctional health nurse is doing a quick assessment on a newly admitted inmate who is HIV positive. Which of the following diseases should the inmate receive screening for immediately? a. Herpes zoster b. Hepatitis B c. Hepatitis C d. Tuberculosis

ANS: D HIV-infected persons, who live near one another, such as in correctional facilities, must be carefully screened and deemed noninfectious before admission to such settings. A person with HIV is more susceptible to opportunistic infections, the most common of which is TB. Hepatitis B and hepatitis C are both transmitted through blood and body fluids which are not as highly contagious as tuberculosis which is transmitted through airborne droplets. Herpes zoster (shingles) is spread by direct contact with fluid from the rash blisters. None of the other options are as directly related to HIV as an opportunistic disease.

A client is being treated for secondary syphilis. Which of the following signs and symptoms would the nurse anticipate the client would exhibit? a. Chancre at the site of entry b. Jaundice c. Difficulty coordinating muscle movements d. Skin rash without itching

ANS: D Secondary syphilis occurs when the organism enters the lymph system and spreads throughout the body. Signs include skin rash on one or more areas of the body and do not cause itching. Other symptoms may include fever, swollen lymph glands, sore throat, patchy hair loss, headaches, weight loss, muscle aches, and fatigue. A chancre at the site of entry is a symptom of primary syphilis. Difficulty coordinating movements is a sign of tertiary syphilis. Jaundice is a sign of congenital syphilis.

A high school student is planning to volunteer at the hospital after school, so he or she needs to have a Mantoux test before beginning. Which of the following information should the nurse provide to the new volunteer? a. "I will be using tiny tines to administer the TB antigen to the skin on your arm." b. "Notify the clinic immediately if you experience any redness or itching at the test site." c. "The areas should be kept dry until you return; cover it with plastic wrap when bathing." d. "You will need to return in 2 to 3 days to have any reaction interpreted."

ANS: D The Mantoux test is a TB skin test that involves a 0.1-ml injection containing 5 tuberculin units of PPD tuberculin (not tines as in the TB tine test). The site should be examined for a reaction 48 to 72 hours (2 to 3 days) after injection. Only induration should be measured, and the results should be recorded in millimeters. The Mantoux test involves a 0.1-ml injection not tines as in the TB tine test. Slight redness or itching at the test site may occur, but that should resolve on its own and does not require the client to notify the clinic. The client can engage in normal activities; the site does not need to be kept dry.

A client was clearly very relieved when an HIV test came back negative. "Thank goodness. I've had sex several times without a condom, and when one of my friends said he was sick, I think I panicked." Which of the following would be most important to emphasize to the client immediately? a. Abstinence is the only way to be certain you are HIV free. b. Sex should be restricted to one partner. c. The test could be wrong and the client might still have an HIV infection. d. The test would not cover any recent infection, so if the client has had recent unprotected sex, the test should be repeated in 3 months.

ANS: D The client needs to understand that the test cannot identify infections that may have been acquired within the previous 3 months before the test. Appearance of the HIV antibody can take up to 12 weeks; thus, this test could be negative now, but positive in 3 months. All persons who have an STD test should be counseled about risk reduction activities before and after the test is done. This information supports the incorrectness of the other options.

Which of the following statements best explains why many health care providers are more afraid of getting hepatitis B (HBV) than HIV? a. Everyone would assume the person infected with hepatitis B is a drug user. b. Having HBV would mean no further employment in health care. c. The fatality rate is higher and occurs sooner with HBV. d. There is no treatment for HBV, which can be a very serious illness.

ANS: D Unlike HIV, there is no treatment for hepatitis B, and although some persons never have symptoms and others fight off the disease, many suffer from chronic hepatitis B, a very serious illness. Others may become chronic carriers of the disease. There are multiple populations that have a high prevalence of HBV including injection drug users, persons with STDs or multiple sex partners, immigrants, and refugees and their descendants who came from areas where there is a high endemic rate of HBV, health care workers, clients on hemodialysis, and inmates of long-term correctional institutions. Those who are infected with HBV are still able to work in health care. There is no evidence that the fatality rate is higher or sooner with HBV than with HIV

A client who is very upset says to the nurse, "But we always used a condom! How could I have genital warts?" Which of the following would be the best response by the nurse? a. "Are you positive you always used a condom?" b. "Condoms don't always work." c. "The condom might have had a tear in the latex." d. "Contact may have occurred outside the area that the condom covers."

ANS: D Warts may grow where barriers, such as condoms, do not cover, and skin-to-skin contact may occur. The challenge of HPV prevention is that condoms do not necessarily prevent infection. Thus, even with correct usage and the use of undamaged condoms, transmission may still occur

Which of the following infectious disease interventions best represents the use of secondary prevention? a. Clients with HIV infection are encouraged to use condoms to protect sexual partners. b. Clients with HIV infection are given medications to improve immunological response. c. Health care workers are encouraged to receive annual vaccinations for influenza. d. Health care workers are required to have a tuberculosis skin test or chest x-ray.

ANS: D - Health care workers are required to have a tuberculosis skin test or chest x-ray. Tuberculosis screening for health care workers is an example of secondary prevention. TB skin tests and chest x-rays are methods of tuberculosis screening. Encouraging clients with HIV to use condoms is an example of primary prevention, because the goal is to prevent exposure to the partner. Encouraging annual influenza vaccinations is also an example of primary prevention. Giving clients with HIV medications is an example of tertiary prevention, because the goal is to reduce complications in those already having the infection.

At a town meeting with public health officials to discuss a communicable disease outbreak, a nurse is asked to explain what is meant by the phrase "a virulent organism." The nurse explains that this means the organism causing the disease is able to do which of the following? a. Bypass normal immunological response mechanisms b. Invade major organ systems c. Produce toxins and poisons that weaken the body d. Produce very severe physical reactions

ANS: D - Produce very severe physical reactions Virulence is the ability to produce a severe pathological reaction.

A student engages in unprotected sex under the influence of alcohol. The student decides to have an HIV test completed the next day. Which of the following results will most likely occur? a. The results will probably be negative for HIV. b. The results will probably be positive for HIV. c. The probability of disease is so low there is no reason to be tested. d. The test results won't be reliable so soon after exposure.

ANS: D - The test results won't be reliable so soon after exposure. It may take up to 6 months after exposure to the HIV virus before an HIV antibody test can test positive, although most infected people will test positive within 3 months. A negative test, therefore, is not a reliable indicator of infection status if exposure is very recent. The incubation period or the time interval between invasion by an infectious agent and the first appearance of signs and symptoms of the disease may be between 10 and 15 years for AIDS.

During an outbreak of hepatitis A, nurses are giving injections of hepatitis A immunoglobulin to selected susceptible persons. Which of the following best describes the type of immunity that will follow the administration of these injections? a. Active immunity b. Long-lasting immunity c. Natural immunity d. Passive immunity

ANS: D - passive community Passive immunity refers to immunization through the transfer of a specific antibody from an immunized individual to a nonimmunized individual, such as the transfer of antibody by administration of an antibody-containing preparation (immune globulin or antiserum). Passive immunity from immune globulin is almost immediate but short-lived. It often is induced as a stopgap measure until active immunity has had time to develop after vaccination.

10. The nurse contacts participants who completed an educational program on breast self-exams to see whether they have any questions and to determine whether they are doing breast self-exams. Which of the following types of evaluation is being implemented by the nurse? A. Final evaluation B. Formative evaluation C. Goal evaluation D. Summative evaluation

ANS: D Summative evaluation Formative evaluation occurs on an ongoing basis while the program exists. In comparison, summative evaluation assesses program outcomes after the program is completed. DIF: Cognitive Level: Apply (Application) REF: pp. 277, 284-285

17. A nurse is conducting a needs assessment but has a limited budget. Which of the following data sources would the nurse most likely eliminate? A. Community forums B. Examination of community indicators C. Focus groups D. Attitude Survey

ANS: D Surveys Attitude scales are probably used most often and are usually phrased in terms of whether a program met its objectives. The client satisfaction survey is an example of an attitude scale often used in the health care delivery system to evaluate the program objectives. Focus groups, community forums, and examination of community indicators are all time consuming and not as directed toward evacuating program goals.

13. A committee concludes that a program's objectives were met and that activities received positive ratings from the community; yet the program will be discontinued because cost was triple the amount anticipated. Which of the following program evaluation measures created a problem? A. Adequacy B. Effectiveness C. Impact D. Sustainability

ANS: D Sustainability The aspects of program evaluation include the following: evaluation of relevance—need for the program; adequacy—program addresses the extent of the need; progress—tracking of program activities to meet program objectives; efficiency—relationship between program outcomes and the resources spent; effectiveness—ability to meet program objectives and the results of program efforts; impact—long-term changes in the client population; and sustainability—enough resources (usually money) to continue the program. DIF: Cognitive Level: Apply (Application) REF: pp. 284-285

5. After completing a needs assessment, the nurse is confident that he has identified the highest priority health programming need within the community. He presents his ideas at a community interest meeting, and the attendees show essentially no interest in being involved. Knowing that the health problem must be addressed he proceeds with implementation as planned. Which of the following is the most likely outcome of the program? A. Community members will become increasingly positive about the new program. B. Others will recognize the importance of the program and become involved. C. The public health agency will both publicize and expand the program. D. The program will fail because of the community's lack of interest.

ANS: D The program will fail because of the community's lack of interest. The needs to be met for the client population must be identified by both the client and the health provider. If the client population does not recognize the need, the program will usually fail. DIF: Cognitive Level: Apply (Application) REF: pp. 278-279

15. Which of the following methods would be the most interactive approach to assessing a community's need? A. Define needs based on the current health status of the community. B. Examine past needs as identified by the agency as well as the community. C. Project future needs based on current trends. D. Use past and current data to project future needs.

ANS: D Use past and current data to project future needs. Stages used in assessing client need include the following: preactive—projecting a future need; reactive—defining the problem based on past needs identified by the client or the agency; inactive—defining the problem based on the existing health status of the population to be served; and interactive—describing the problem using past and present data to project future population needs. DIF: Cognitive Level: Understand (Comprehension) REF: p. 278

6. A nurse is assessing a community to determine the feasibility of implementing a new program on bike safety for youth in the community. Which of the following aspects should the nurse investigate to make this determination? A. Whether the community, especially agency clients, desire a program B. Whether local politicians support the agency's idea for a program C. Whether agency professionals think a program is needed D. Whether all involved support the need for such a program

ANS: D Whether all involved support the need for such a program What people think about the need for a program, or program feasibility, might differ among health providers, agency administrators, policymakers, and potential clients. Feasibility means the program's viability, practicality, achievability, or likelihood of success. Everyone involved must be supportive for a program to succeed. DIF: Cognitive Level: Apply (Application) REF: pp. 278-279

A community-based HIV/AIDS clinic would be concerned about which aspects of the Public Health Code of Ethics? Select all that apply. A. Autonomy of the professional. B. Confidentiality, when possible. C. Funding. D. Advocacy for disenfranchised persons. E. Respect of only community rights.

B and D

A community-oriented nurse understands that the most important use of the principles for effective advocacy is to: A. Act as a friend and advocate for the target population served to promote research and data gathering. B. Act in the community's best interest, in keeping with community direction, while maintaining diligence, ongoing communication, and confidentiality. C. Ensure that public funds are spent only for the priorities stated in the legislation that authorized program funding and promote regulations and legislation. D. Give more priority to the opinions and feelings of key community leaders than to data when determining program priorities.

B. Act in the community's best interest, in keeping with community direction, while maintaining diligence, ongoing communication, and confidentiality. The six principles of effective advocacy include acting in the community's best interest, acting in accordance with the community's wishes, keeping the community informed, acting with diligence, maintaining impartiality, and maintaining confidentiality. Bateman's practical framework for advocacy places the advocate's core skills within the context of the six principles of effective advocacy.Cognitive Level: AnalysisAssociated Chapter Objective: 6Page Reference: 136-137

A nurse has worked for years in an intensive care unit and decides to take a position as a community health nurse directing a local public health unit. Her first assignment is to perform an assessment of the community's migrant farmworker population. The nurse's most appropriate next step is to: A. Begin the community assessment using a survey tool proven effective in previous public health unit assessments. B. Perform a literature review to study assessment data for similar populations. C. Enroll in a community health nursing graduate course to learn community research, measurement, and analysis techniques. D. Form a relationship with the farmworkers' community leaders and other key informants.

C. Enroll in a community health nursing graduate course to learn community research, measurement, and analysis techniques. Two core functions of community-oriented nursing are assessment and assurance. Both of these core functions have their foundation in the ethical tenets of competency. Nurses assigned to develop community knowledge must be adequately prepared to collect data on groups and populations. The techniques employed differ from those used when caring for individuals. Use of the wrong research techniques leads to wrong assessments, with the potential for developing interventions that harm rather than help the target population.Cognitive Level: SynthesisAssociated Chapter Objective: 4Page Reference: 133-134

Caring and the ethic of care are core values of community health nursing. This ethical view was developed in the mid-1980s and early 1990s in response to the technical advances in the health care sciences and the desire of nurses to: A. Apply gender-related voices to moral judgment. B. Apply principles of utilitarianism. C. Differentiate distributive justice from beneficence. D. Differentiate nursing practice from medical practice.

D. Differentiate nursing practice from medical practice. The view of caring and the ethic of care that emerged in the mid-1980s and early 1990s was a response to technological advances in science and the desire of nurses to differentiate nursing practice from medical practice. It is closely related to feminist ethics.Cognitive Level: AnalysisAssociated Chapter Objective: 3Page Reference: 131-132

Although nursing has a strong implicit heritage of ethical values and morality, it was not until the 1960s that several seminal events significantly shifted the focus to ethical decision-making processes. One of those seminal events was the emergence of the field of bioethics and the other event was: A. Adoption of the American Nurses Association (ANA)'s Code for Professional Nurses. B. Adoption of the American Public Health Association's Public Health Code of Ethics. C. Adoption of the International Council of Nurses' Code of Ethics for Nursing. D. Issuance of the ANA's position statement on nursing education in institutions of higher education.

D. Issuance of the ANA's position statement on nursing education in institutions of higher education. In the 1960s, the ANA recommended that all nursing education occur in institutions of higher education; a shift thereby occurred from reliance on inherent ethical tenets and values to the explicit study of ethics and ethical decision-making processes. This, in conjunction with the emergence of the field of bioethics, altered the curriculum content of nursing education programs.Cognitive Level: AnalysisAssociated Chapter Objective: 1Page Reference: 126

A low-vision client with very early dementia takes pride in her independence. The client, who lives alone in an apartment, tells her nurse she has always enjoyed using fragranced candles. The most appropriate intervention for the nurse is to apply the principle of: A. Autonomy, in which the nurse leaves the matches and candles accessible to the client. B. Beneficence, in which the nurse maintains the client's dignity and reinforces to the client how pleasant it is to use scented candles. C. Nonmaleficence, in which the nurse counsels the client regarding the dangers of the use of candles and matches by someone with low vision. D. Respect for autonomy, in which the nurse recommends to the client an arrangement whereby the caregiver lights the candles when the caregiver is present in the home and removes the matches and candles from the kitchen cabinet.

D. Respect for autonomy, in which the nurse recommends to the client an arrangement whereby the caregiver lights the candles when the caregiver is present in the home and removes the matches and candles from the kitchen cabinet. The nurse applies the principle of respect for autonomy. The nurse wants to maintain the client's dignity and quality of life, and to help the client be as independent as possible. But at the same time, the nurse must choose actions that reduce the risk of harm to others.Cognitive Level: ApplicationAssociated Chapter Objective: 3Page Reference: 129-130

The new director of a local public health unit reviews the personnel records of the unit staff and notes that none of the assigned public health nurses has academic training in community and public health nursing. The director's next step should be to: A. Recognize that staff members have learned on the job through years of experience and that they will be able to teach their new director basic community and public health principles. B. Make arrangements to hire new staff with appropriate formal training. C. Realize that with the current nursing shortage the unit should be grateful to have long-term staff. D. Review continuing education records to determine if the nurses have received training in community-oriented nursing and ethics, and plan for any needed training.

D. Review continuing education records to determine if the nurses have received training in community-oriented nursing and ethics, and plan for any needed training.


Set pelajaran terkait

Chapter 23: Neurocognitive Disorders

View Set

Risk Analysis Oral Exam Questions

View Set